RE: [obm-l] IMO Polinomio irredutivel

2010-06-25 Por tôpico Luís Lopes

Sauda,c~oes, 

 

b) a_0=5. Irredutível (por Eisenstein com p=5)
 
c) a_0=6. Redutível. ??

 

x^2 + 5x + 6 = (x+2)(x+3)

x^3 + 5x^2 + 6 = ?? (*)

Mostre que se m é composto a fatoração de polinômios 
mód. m não é única. (**) 

 

Como provar (**) e depois usar em (*) ? 

 

[]'s 

Luís 


 


 



From: qed_te...@hotmail.com
To: obm-l@mat.puc-rio.br
Subject: [obm-l] IMO Polinomio irredutivel
Date: Thu, 24 Jun 2010 22:20:17 +



Sauda,c~oes, oi Johann Dirichlet, 
 
Fiz reply e a mensagem não foi. Mando como nova msg. 

Vc(s) saberia dizer o ano da IMO deste problema? 
Haveria uma outra solução para este problema? 
 
O mesmo problema x^n + 5x^{n-1} + a_0 para 
o termo independente a_0 igual a 4, 5 e 6. 
 
a) a_0=4. Redutível para n par pois 1 - 5 + 4 = 0
 
b) a_0=5. 
 
c) a_0=6. 
 
Juntamente com o problema 
 
Mostre que se m é composto a fatoração de polinômios 
mód. m não é única. 
 
[]s 
Luís 




VEJA TODOS OS SEUS EMAILS DE VÁRIAS CONTAS COM UM SÓ LOGIN. CLIQUE AQUI E VEJA 
COMO.
  
_
VEJA SEUS EMAILS ONDE QUER QUE VOCÊ ESTEJA, ACESSE O HOTMAIL PELO SEU CELULAR 
AGORA.
http://celular.windowslive.com.br/hotmail.asp?produto=Hotmailutm_source=Live_Hotmailutm_medium=Taglineutm_content=VEJASEUSEM84utm_campaign=MobileServices

[obm-l] IMO Polinomio irredutivel

2010-06-24 Por tôpico Luís Lopes

Sauda,c~oes, 

 

Na página 27 do livro 21 Aulas de Mat. Olímp. do C. Y. Shine 

encontro o seguinte problema: 

 

Prove que o polinômio x^n + 5x^{n-1} + 3 é irredutível em Q(Z). 

 

Gostaria de ver a solução baseada com o que foi mostrado no 

livro e as referências (fonte e solução) da página do kalva. 

 

Aliás o link para a homepage do kalva do site da Olimp. Paulista de Mat. 

estava quebrado quando tentei entrar. 

 

Obrigado. 

 

[]'s 

Luís 

 
  
_
VEJA SEUS EMAILS ONDE QUER QUE VOCÊ ESTEJA, ACESSE O HOTMAIL PELO SEU CELULAR 
AGORA.
http://celular.windowslive.com.br/hotmail.asp?produto=Hotmailutm_source=Live_Hotmailutm_medium=Taglineutm_content=VEJASEUSEM84utm_campaign=MobileServices

Re: [obm-l] IMO Polinomio irredutivel

2010-06-24 Por tôpico Johann Dirichlet
O site do Scholes morreu :(
Tente pelo Archive.org.

A solucao que eu conheco e mais ou menos essa:
Este polinomio nao tem raizes racionais (é só testar 1,3,-1 e -3 que
seriam as possibilidades).

Modulo 3, esse polinomio fatora como x^(n-1)(x+5).

Se pudermos escrever isto como P(x)Q(x), teremos
P(x)Q(x)=x^(n-1)(x+5). Logo P=x^k(x+5)+3X e Q=x^l+3Y com k+l=n-1 e X,Y
polinomios. Daí dá para obter uam contradicao simplesmente
multiplicando os polinomios.

Em 24 de junho de 2010 11:39, Luís Lopes qed_te...@hotmail.com escreveu:
 Sauda,c~oes,

 Na página 27 do livro 21 Aulas de Mat. Olímp. do C. Y. Shine
 encontro o seguinte problema:

 Prove que o polinômio x^n + 5x^{n-1} + 3 é irredutível em Q(Z).

 Gostaria de ver a solução baseada com o que foi mostrado no
 livro e as referências (fonte e solução) da página do kalva.

 Aliás o link para a homepage do kalva do site da Olimp. Paulista de Mat.
 estava quebrado quando tentei entrar.

 Obrigado.

 []'s
 Luís


 
 O SEU NAVEGADOR PODE TE PROTEGER DE FRAUDES NA WEB. VEJA DICAS DE INTERNET
 EXPLORER 8



-- 
/**/
Quadrinista e Taverneiro!

http://tavernadofimdomundo.blogspot.com  Histórias, Poemas, Quadrinhos e Afins
http://baratoeletrico.blogspot.com / Ativismo Digital (?)
http://bridget-torres.blogspot.com/  Personal! Do not edit!

=
Instru��es para entrar na lista, sair da lista e usar a lista em
http://www.mat.puc-rio.br/~obmlistas/obm-l.html
=


Re: [obm-l] IMO Polinomio irredutivel

2010-06-24 Por tôpico Johann Dirichlet
Ah, o site:
http://www.cs.cornell.edu/~asdas/IMO/imo.html

Uma versao antiga.

Em 24 de junho de 2010 12:24, Johann Dirichlet
peterdirich...@gmail.com escreveu:
 O site do Scholes morreu :(
 Tente pelo Archive.org.

 A solucao que eu conheco e mais ou menos essa:
 Este polinomio nao tem raizes racionais (é só testar 1,3,-1 e -3 que
 seriam as possibilidades).

 Modulo 3, esse polinomio fatora como x^(n-1)(x+5).

 Se pudermos escrever isto como P(x)Q(x), teremos
 P(x)Q(x)=x^(n-1)(x+5). Logo P=x^k(x+5)+3X e Q=x^l+3Y com k+l=n-1 e X,Y
 polinomios. Daí dá para obter uam contradicao simplesmente
 multiplicando os polinomios.

 Em 24 de junho de 2010 11:39, Luís Lopes qed_te...@hotmail.com escreveu:
 Sauda,c~oes,

 Na página 27 do livro 21 Aulas de Mat. Olímp. do C. Y. Shine
 encontro o seguinte problema:

 Prove que o polinômio x^n + 5x^{n-1} + 3 é irredutível em Q(Z).

 Gostaria de ver a solução baseada com o que foi mostrado no
 livro e as referências (fonte e solução) da página do kalva.

 Aliás o link para a homepage do kalva do site da Olimp. Paulista de Mat.
 estava quebrado quando tentei entrar.

 Obrigado.

 []'s
 Luís


 
 O SEU NAVEGADOR PODE TE PROTEGER DE FRAUDES NA WEB. VEJA DICAS DE INTERNET
 EXPLORER 8



 --
 /**/
 Quadrinista e Taverneiro!

 http://tavernadofimdomundo.blogspot.com  Histórias, Poemas, Quadrinhos e 
 Afins
 http://baratoeletrico.blogspot.com / Ativismo Digital (?)
 http://bridget-torres.blogspot.com/  Personal! Do not edit!




-- 
/**/
Quadrinista e Taverneiro!

http://tavernadofimdomundo.blogspot.com  Histórias, Poemas, Quadrinhos e Afins
http://baratoeletrico.blogspot.com / Ativismo Digital (?)
http://bridget-torres.blogspot.com/  Personal! Do not edit!

=
Instru��es para entrar na lista, sair da lista e usar a lista em
http://www.mat.puc-rio.br/~obmlistas/obm-l.html
=


[obm-l] IMO Polinomio irredutivel

2010-06-24 Por tôpico Luís Lopes

Sauda,c~oes, oi Johann Dirichlet, 

 

Fiz reply e a mensagem não foi. Mando como nova msg. 

Vc(s) saberia dizer o ano da IMO deste problema? 
Haveria uma outra solução para este problema? 
 
O mesmo problema x^n + 5x^{n-1} + a_0 para 
o termo independente a_0 igual a 4, 5 e 6. 
 
a) a_0=4. Redutível para n par pois 1 - 5 + 4 = 0
 
b) a_0=5. 
 
c) a_0=6. 
 
Juntamente com o problema 
 
Mostre que se m é composto a fatoração de polinômios 
mód. m não é única. 
 
[]s 
Luís 

  
_
QUER FICAR SEMPRE EM CONTATO COM SEUS AMIGOS? ACESSE O MESSENGER PELO SEU 
CELULAR.
http://celular.windowslive.com.br/messenger.asp?produto=Messengerutm_source=Live_Hotmailutm_medium=Taglineutm_content=QUERFICARS82utm_campaign=MobileServices

[obm-l] IMO 2008 Segunda Questão

2009-02-08 Por tôpico jjunior
Está correta a solução para o problema 2 da IMO de 2008 aqui não reproduzido?

a)  Se x.y.z =1, então, um ou dois desses números (e não os três
simultaneamente) terá módulo maior que um. Eles são distintos de um por
hipótese.  Assim, para esse número com tal módulo (ou para esses dois
números), a razão entre o seu quadrado e o seu quadrado diminuído de um é
superior a um. Logo, a soma dos três termos, sempre positivos, é superior
a 1, o que prova a.

b) Sem perda de generalidade, x = a/b, y = b/c e z = c/a, com a, b e c
pertencentes aos inteiros. a, b e c são diferentes entre si dois a dois,
já que x, y, e z são distintos de 1. Então, se se provar que [a/(a-b)]^2 +
[b/(b-c)]^2 + [c/(c-a)]^2 é igual a 1, acaba a questão. Tem muito braço
nessa conta.

   Para facilitar a escrita, adotar-se-á a seguir que a.a = a2, e o dobro
de a como 2a. Logo, a2/(a-b)2 + b2/(b-c)2 + c2/(c-a)2 = [a2(b-c)2(c-a)2
+ b2(a-b)2(c-a)2 + c2(a-b)2(b-c)2]/(a-b)2(b-c)2(c-a)2.

   O numerador da razão última acima é: (a2-2ab+b2)(b2-2bc+c2)(c2-2ac+a2)
= a2b2c2 + a2b2(-2ac) + a2b2a2 + a2(-2bc)c2 + a2(-2bc)(-2ac) +
a2(-2bc)a2 + a2c2c2 + a2c2(-2ac) + a2c2a2 + (-2ab)b2c2 + (-2ab)b2(-2ac)
+ (-2ab)b2a2 + (-2ab)(-2bc)c2 + (-2ab)(-2bc)(-2ac) + (-2ab)(-2bc)a2 +
(-2ab)c2c2 + (-2ab)c2(-2ac) + (-2ab)c2a2 + b2b2c2 + b2b2(-2ac) + b2b2a2
+ b2(-2bc)c2 + b2(-2bc)(-2ac)+ b2(-2bc)(a2)+b2c2c2 + b2c2(-2ac) +
b2c2a2.

   Já o denominador (é de se esperar que seja idêntico ao numerador,
deseja-se) é [a2(b2-2bc+c2)(c2-2ac+a2) + b2(a2-2ab+b2)(c2-2ac+a2) +
c2(a2-2ab+b2)(b2-2bc+c2) = a2b2c2 + a2b2(-2ac) + a2b2a2 + a2(-2bc)c2 +
a2(-2bc)(-2ac) + a2(-2bc)a2 + a2c2c2 + a2c2(-2ac) +a2c2a2 + b2a2c2 +
b2a2(-2ac) + b2a2a2 + b2(-2ab)c2 + b2(-2ab)(-2ac) + b2(-2ab)a2 + b2b2c2
+ b2b2(-2ac) + b2b2a2 + c2a2b2 + c2a2(-2bc) + c2a2c2 + c2(-2ab)b2 +
c2(-2ab)(-2bc) + c2(-2ab)c2 + c2b2b2 + c2b2(-2bc) + c2b2c2.

   Bem, com uma comparação atenciosa (não intelectual), percebe-se a
igualdade desejada, o que prova b.


=
Instruções para entrar na lista, sair da lista e usar a lista em
http://www.mat.puc-rio.br/~obmlistas/obm-l.html
=


RE: [obm-l] IMO

2008-05-15 Por tôpico pedro barboza

encontrei uma solução para essa questão na eureka n°17, ela está disponível no 
site da obm


 From: [EMAIL PROTECTED]
 To: obm-l@mat.puc-rio.br
 Subject: [obm-l] IMO
 Date: Tue, 6 May 2008 22:52:42 -0300
 
 
 Quem puder me ajudar eu agradeço muitíssimo!
 
 
 
 “Os lados AB e AC de um triângulo ABC tangenciam uma circunferência de centro 
 O em E e F, respectivamente. A projeção ortogonal do centro sobre BC 
 determina em BC o ponto J. O prolongamento de OJ “cruza” EF em D. Seja M o 
 ponto médio de BC, prove que os pontos A, D e M são colineares.”
 
 
 
 Desde já agradeço!
 
 
 
 JG
 
 
 No virus found in this outgoing message.
 Checked by AVG.
 Version: 7.5.524 / Virus Database: 269.23.9/1416 - Release Date: 05/05/2008 
 17:11

_
Receba GRÁTIS as mensagens do Messenger no seu celular quando você estiver 
offline. Conheça  o MSN Mobile!
http://mobile.live.com/signup/signup2.aspx?lc=pt-br
=
Instruções para entrar na lista, sair da lista e usar a lista em
http://www.mat.puc-rio.br/~obmlistas/obm-l.html
=


[obm-l] IMO

2008-05-06 Por tôpico João Gabriel Preturlan
Quem puder me ajudar eu agradeço muitíssimo!

 

“Os lados AB e AC de um triângulo ABC tangenciam uma circunferência de
centro O em E e F, respectivamente. A projeção ortogonal do centro sobre BC
determina em BC o ponto J. O prolongamento de OJ “cruza” EF em D. Seja M o
ponto médio de BC, prove que os pontos A, D e M são colineares.”

 

Desde já agradeço!

 

JG


No virus found in this outgoing message.
Checked by AVG. 
Version: 7.5.524 / Virus Database: 269.23.9/1416 - Release Date: 05/05/2008
17:11
 


Re: [obm-l] IMO 2007

2007-08-16 Por tôpico fernandobarcel
Oi João Carlos,
visitei o site da IMO e no fórum aparecem ao todo 4 soluções para este problema.
E são muito parecidas com a do Ponce.
Teve um email nervoso que você sugeriu que ele podia continuar de onde você 
parou. Sinceramente, 
nem que o cara fosse mágico, porque esse caminho da gente tá errado no primeiro 
passo mesmo.
A continuação seria jogar tudo no lixo, e voltar pro início, só que ele não 
falou.
Eu também tentei resolver dividindo tudo ao meio, mas cheguei a conclusão que 
não dá certo.
Olha o fórum da IMO onde tratam dessa questão em 
http://www.mathlinks.ro/viewtopic.php?p=893746#893746

Agora, que coisa esse teu surto! cara, isso é uísque do paraguay!


-- Início da mensagem original ---


  De: [EMAIL PROTECTED]
Para: obm-l@mat.puc-rio.br
  Cc: 
Data: Fri, 27 Jul 2007 21:57:33 -0400
 Assunto: Re: [obm-l] IMO 2007

 
Alguém, por gentileza, comente o surto abaixo. Ponce, preliminarmente,
creio que está correto. Vou olhar com maior atenção.
 
O surto:
 
Vamos busca modelar (como se modela argila) esse conjunto
competição.
 
Não estou brincando não, falo sério.
 
Cada conjunto clique desse é um monte de argila. Existe um
conjunto maior com 2n elementos.
 
Esses conjuntos de barro podem estar unidos. Essas uniões
são as amizades que ligam os conjuntos clique sem transformá-los num
conjunto clique maior. Também podem existir montes sem ligação com
nenhum outro.
 
Ora, sempre é possível dividir todo o conjunto competição,
de forma que o maior conjunto clique com 2n participantes seja divido
ao meio e os demais também ao meio (se par) ou em dois números
inteiros e consecutivos (se ímpares) e, sem tanta preocupação com as
amizades inter-cliques, pois elas não aumentam o tamanho de cada
conjunto. Assim, sempre será possível se ter aí o que se deseja
provar.
 
Falta precisão, claro, mais essa pode ser simples a partir
da idéia acima, creio.
 
 
Fraternalmente, João.
 

 
Ola' Shine, Joao e colegas da lista,
acho que eu poderia melhorar a explicacao, mas vamos la' assim
mesmo...
Sempre podemos dividir os competidores da seguinte forma:
Coloque o maior clique na sala A e todos os outros na sala B.
Se na sala B tambem houver um clique com o tamanho da sala A, a
divisao esta' completa. Se nao, execute a etapa X.
Etapa X :
Passe um competidor da sala A para a sala B.
Dessa forma, o clique em A diminui de 1 unidade, alguns cliques em
B crescem de 1 unidade, e outros cliques em B nao se alteram.
Entao:
- Se o(s) maior(es) clique(s) em B ainda nao igualou o clique em
A, repita a etapa X.
- Se o(s) maior(es) clique(s) em B igualou o clique em A, a
divisao esta' completa.
- E se o(s) maior(es) clique(s) em B ultrapassou o clique em A ?
Bem, em cada um desses cliques (o clique formado pelos migrados de A
nao esta' entre estes cliques, pois o clique original em A era par),
existe algum competidor que nao estava originalmente em A .
Passe esse competidor para A (faca isso em todos os cliques de B
que ultrapassaram o valor em A).
Agora a divisao esta'  completa.
OBS: Poderia acontecer de todos os jogadores transferidos para A
formarem um clique independente, superior ao clique em A ?
Nao, caso contrario eles ja' estariam formando um clique na sala B
igual ao clique em A, antes da ultima passagem de alguem de A para
B, e o processo ja' teria terminado.
Note que o clique original em A e' par. Assim, todo o processo
descrito termina no maximo quando metade dos competidores em A tiver
sido transferida para B.
[]'s
Rogerio Ponce
Carlos Yuzo Shine [EMAIL PROTECTED] escreveu:
3. Numa competição de matemática, alguns competidores são amigos.
Amizade é sempre mútua. Chame um grupo de competidores de clique se
quaisquer dois entre eles são amigos. Em particular, qualquer grupo
com menos de dois amigos é um clique. O número de membros de um clique
é o seu tamanho.
Dado que, nesta competição, o maior tamanho de um clique é par, prove
que os competidores podem ser divididos em duas salas tais que o maior
tamanho de um clique em uma sala é igual ao maior tamanho de um clique
na outra sala.
[]'s
Shine
Alertas do Yahoo! Mail em seu celular. [1]Saiba mais.
==
==Instruções para entrar na lista, sair da lista e usar a lista em
http://www.mat.puc-rio.br/~nicolau/olimp/obm-l.html
==
==
 


=
Instruções para entrar na lista, sair da lista e usar a lista em
http://www.mat.puc-rio.br/~nicolau/olimp/obm-l.html
=


Re: [obm-l] IMO 2007

2007-08-02 Por tôpico JoaoCarlos_Junior
Se a amizade não existia no conjunto competição, então, ela não passará a existir nas salas. Uma amizade é restabelecida se os recíprocos amigos forem inclusos na mesma sala, mesmo que em momentos distintos.Sim de fato, a amizade somente ficará quebrada (cortada, como queira) somente se os amigos estiverem nas salas distintas.Podemos continuar a escrever algo (que, a meu ver, está cada vez mais próximo de uma resposta integralmente satisfatória) na linguagem da própria pergunta, porém, devemos ser agradecidos com o êxtase - princípio do auxílio ? que nos conduziu ou quer nos conduzir à resposta, por analogia. Prefiro a segunda à primeira. Permita-me, assim, expressar-me. Passar da linguagem em analogia à do próprio problema me não parece difícil. Então:1) Da massa de argila (toda a competição), podemos separar dela o conjunto clique máximo integralmente. Empós, dividimo-lo no meio, jogando cada metade em duas mesas distintas (as salas).2) Os montes menores também devem ser divididos em duas partes, de forma que cada uma dessas partes cliques sejam de tamanho menor que as metades acima (no máximo, há uma igualdade, não é difícil verificar). Percebamos que se havia anteriormente amizade entre os elementos desses conjuntos menores entre eles próprios e deles para com os elementos do conjunto maior, as amizades ficarão restabelecidas entre os elementos que já eram previamente amigos, porém, agora, só para aqueles que estão na mesma sala. Esses restabelecimentos, no entanto, não aumentam os tamanhos dos conjuntos cliques cortados.3) depois a massa que sobrou você pode cortá-la ou não (como queira), jogando-a integralmente em uma só sala, ou retalhá-la a gosto, lançando as partes em ambas as salas, sob qualquer critério.Com sinceridade, sem o menor grau de sofisma: agrado tuas contraposições, Ponce, que me impeliram à frente nessa resolução. Se ainda houver alguma(s), por gentileza principalmente a mim, manifeste-a(s).Desculpe-me não ter respondido logo. Obrigado.Fraternalmente, João. [EMAIL PROTECTED] escreveu: -Para: obm-l@mat.puc-rio.brDe: Rogerio Ponce [EMAIL PROTECTED]Enviado por: [EMAIL PROTECTED]Data: 28/07/2007 4:09Assunto: Re: [obm-l] IMO 2007Ola' Joao, suponha a competicao com os competidores numerados de 1 a 13, formando os seguintes cliques: 1, 2, 3, 4 5, 6, 7 8, 9, 10 11, 12, 13 5, 8, 9 5, 8, 11 5, 9, 12 6, 7, 10 7, 9, 10 7, 11, 13 Repare que nao da' para pensarmos em dividir cada conjunto ao meio (ou proximo do meio) de forma independente, pois eles nao sao obrigatoriamente disjuntos. Entao, quando voce faz a divisao de um clique, muitas vezes tambem estara separando (ou agrupando) outro clique. Assim, embora o maior clique tenha inicialmente "2n" elementos , nao e' verdade que a sua forma de dividi-los va' produzir cliques com no maximo "n" elementos (embora essa seja a nossa primeira impressao). Seguindo sua sugestao, poderiamos separar os competidores assim: [1, 2] na sala "A" , [3, 4] na sala "B" [5, 6] na sala "A" , [7] na sala "B" [8, 9] na sala "A" , [10] na sala "B" [11, 12] na sala "A" , [13] na sala "B" Parariamos a divisao neste ponto, uma vez que ja' teriamos dado destino a todos os competidores. Entretanto, na sala "A" existe um clique (5,8,11) com 3 competidores , enquanto os maiores cliques da sala "B" nao passam de 2 competidores. Acho que este exemplo serve de partida para voce elaborar o que pode acontecer durante qualquer outra forma de divisao. []'s Rogerio Ponce -- Alguém, por gentileza, comente o surto abaixo. Ponce, preliminarmente, creio que está correto. Vou olhar com maior atenção. O surto:   Vamos busca modelar (como se modela argila) esse conjunto competição.  Não estou brincando não, falo sério.  Cada conjunto clique desse é um monte de argila. Existe um conjunto maior com 2n elementos.  Esses conjuntos de barro podem estar unidos. Essas uniões são as amizades que ligam os conjuntos clique sem transformá-los num conjunto clique maior. Também podem existir montes sem ligação com nenhum outro.  Ora, sempre é possível dividir todo o conjunto competição, de forma que o maior conjunto clique com 2n participantes seja divido ao meio e os demais também ao meio (se par) ou em dois números inteiros e consecutivos (se ímpares) e, sem tanta preocupação com as amizades inter-cliques, pois elas não aumentam o tamanho de cada conjunto. Assim, sempre será possível se ter aí o que se deseja provar.  Falta precisão, claro, mais essa pode ser simples a partir da idéia acima, creio.  Fraternalmente, João. Alertas do Yahoo! Mail em seu celular. Saiba mais . Instruções para entrar na lista, sair da lista e usar a lista em
http://www.mat.puc-rio.br/~nicolau/olimp/obm-l.html
===

Re: [obm-l] IMO 2007

2007-07-28 Por tôpico Rogerio Ponce
Ola' Joao,
suponha a competicao com os competidores numerados de 1 a 13, formando os 
seguintes cliques:
1, 2, 3, 4
5, 6, 7
8, 9, 10
11, 12, 13
5, 8, 9
5, 8, 11
5, 9, 12
6, 7, 10
7, 9, 10
7, 11, 13

Repare que nao da' para pensarmos em dividir cada conjunto ao meio (ou proximo 
do meio) de forma independente, pois eles nao sao obrigatoriamente disjuntos. 
Entao, quando voce faz a divisao de um clique, muitas vezes tambem estara 
separando (ou agrupando) outro clique.

Assim, embora o maior clique tenha inicialmente 2n elementos , nao e' verdade 
que a sua forma de dividi-los va' produzir cliques com no maximo n elementos 
(embora essa seja a nossa primeira impressao).

Seguindo sua sugestao, poderiamos separar os competidores assim:
[1, 2] na sala A ,  [3, 4] na sala B
[5, 6]  na sala A ,  [7] na sala B
[8, 9] na sala A ,  [10] na sala B
[11, 12] na sala A ,   [13] na sala B

Parariamos a divisao neste ponto, uma vez que ja' teriamos dado destino a todos 
os competidores.
Entretanto, na sala A existe um clique (5,8,11) com 3 competidores , enquanto 
os maiores cliques da sala B nao passam de 2 competidores.

Acho que este exemplo serve de partida para voce elaborar o que pode acontecer 
durante qualquer outra forma de divisao.

[]'s
Rogerio Ponce

--
 
 Alguém, por gentileza, comente o surto abaixo. Ponce, preliminarmente, creio 
que está correto. Vou olhar com maior atenção.
  O surto: 
  Vamos busca modelar (como se modela argila) esse conjunto 
competição. 
  Não estou brincando não, falo sério.
  Cada conjunto clique desse é um monte de argila. Existe um 
conjunto maior com 2n elementos. 
  Esses conjuntos de barro podem estar unidos. Essas uniões são as 
amizades que ligam os conjuntos clique sem transformá-los num conjunto clique 
maior. Também podem existir montes sem ligação com nenhum outro.
  Ora, sempre é possível dividir todo o conjunto competição, de 
forma que o maior conjunto clique com 2n participantes seja divido ao meio e os 
demais também ao meio (se par) ou em dois números inteiros e consecutivos (se 
ímpares) e, sem tanta preocupação com as amizades inter-cliques, pois elas não 
aumentam o tamanho de cada conjunto. Assim, sempre será possível se ter aí o 
que se deseja provar.
  Falta precisão, claro, mais essa pode ser simples a partir da 
idéia acima, creio.
   
  Fraternalmente, João.
 


   Alertas do Yahoo! Mail em seu celular. Saiba mais.

Re:Res:[obm-l] IMO 2007

2007-07-27 Por tôpico JoaoCarlos_Junior
Acho que você está certo, vou analisar.[EMAIL PROTECTED] escreveu: -Para: "obm-l" obm-l@mat.puc-rio.brDe: "fernandobarcel" [EMAIL PROTECTED]Enviado por: [EMAIL PROTECTED]Data: 26/07/2007 21:53Assunto: Re:Res:[obm-l] IMO 2007João,"clique é um grupo de competidores onde quaisquer dois entre eles são amigos".Portanto, a competição pode não ser um clique.Abraços,-- Início da mensagem original ---  Tentativa ao terceiro problema A própria competição (que encerra todos os competidores) é clique, pois : 1) Há alguns competidores amigos; 2) A amizade é mútua, então, há pelo menos  dois amigos na competição.  ... [EMAIL PROTECTED] escreveu: -   3. Numa competição de matemática, alguns competidores são amigos.   Amizade é sempre mútua. Chame um grupo de competidores de clique se   quaisquer dois entre eles são amigos. Em particular, qualquer grupo   com menos de dois amigos é um clique. O número de membros de um   clique é o seu tamanho.   Dado que, nesta competição, o maior tamanho de um clique é par,   prove que os competidores podem ser divididos em duas salas tais   que o maior tamanho de um clique em uma sala é igual ao maior   tamanho de um clique na outra sala.=Instruções para entrar na lista, sair da lista e usar a lista emhttp://www.mat.puc-rio.br/~nicolau/olimp/obm-l.html=Instruções para entrar na lista, sair da lista e usar a lista em
http://www.mat.puc-rio.br/~nicolau/olimp/obm-l.html
===

Re: [obm-l] IMO 2007

2007-07-27 Por tôpico Rogerio Ponce
Ola' Shine, Joao e colegas da lista,
acho que eu poderia melhorar a explicacao, mas vamos la' assim mesmo...

Sempre podemos dividir os competidores da seguinte forma:

Coloque o maior clique na sala A e todos os outros na sala B.
Se na sala B tambem houver um clique com o tamanho da sala A, a divisao 
esta' completa. Se nao, execute a etapa X.

Etapa X :

Passe um competidor da sala A para a sala B.

Dessa forma, o clique em A diminui de 1 unidade, alguns cliques em B 
crescem de 1 unidade, e outros cliques em B nao se alteram.

Entao:
- Se o(s) maior(es) clique(s) em B ainda nao igualou o clique em A, repita 
a etapa X.

- Se o(s) maior(es) clique(s) em B igualou o clique em A, a divisao esta' 
completa.

- E se o(s) maior(es) clique(s) em B ultrapassou o clique em A ?

Bem, em cada um desses cliques (o clique formado pelos migrados de A nao 
esta' entre estes cliques, pois o clique original em A era par), existe algum 
competidor que nao estava originalmente em A .

Passe esse competidor para A (faca isso em todos os cliques de B que 
ultrapassaram o valor em A).

Agora a divisao esta'  completa.

OBS: Poderia acontecer de todos os jogadores transferidos para A formarem um 
clique independente, superior ao clique em A ?

Nao, caso contrario eles ja' estariam formando um clique na sala B igual ao 
clique em A, antes da ultima passagem de alguem de A para B, e o processo 
ja' teria terminado.

Note que o clique original em A e' par. Assim, todo o processo descrito 
termina no maximo quando metade dos competidores em A tiver sido transferida 
para B.


[]'s
Rogerio Ponce



Carlos Yuzo Shine [EMAIL PROTECTED] escreveu:
3. Numa competição de matemática, alguns competidores são amigos. Amizade é 
sempre mútua. Chame um grupo de competidores de clique se quaisquer dois entre 
eles são amigos. Em particular, qualquer grupo com menos de dois amigos é um 
clique. O número de membros de um clique é o seu tamanho.

Dado que, nesta competição, o maior tamanho de um clique é par, prove que os 
competidores podem ser divididos em duas salas tais que o maior tamanho de um 
clique em uma sala é igual ao maior tamanho de um clique na outra sala.

[]'s
Shine




   Alertas do Yahoo! Mail em seu celular. Saiba mais.

Re: [obm-l] IMO 2007

2007-07-27 Por tôpico JoaoCarlos_Junior

Alguém, por gentileza, comente o surto abaixo. Ponce, preliminarmente, creio que está correto. Vou olhar com maior atenção.
O surto:  
 Vamos busca modelar (como se modela argila) esse conjunto competição. 
 Não estou brincando não, falo sério.
 Cada conjunto clique desse é um monte de argila. Existe um conjunto maior com 2n elementos. 
 Esses conjuntos de barro podem estar unidos. Essas uniões são as amizades que ligam os conjuntos clique sem transformá-los num conjunto clique maior. Também podem existir montes sem ligação com nenhum outro.
 Ora, sempre é possível dividir todo o conjunto competição, de forma que o maior conjunto clique com 2n participantes seja divido ao meio e os demais também ao meio (se par) ou em dois números inteiros e consecutivos (se ímpares) e, sem tanta preocupação com as amizades inter-cliques, pois elas não aumentam o tamanho de cada conjunto. Assim, sempre será possível se ter aí o que se deseja provar.
 Falta precisão, claro, mais essa pode ser simples a partir da idéia acima, creio.

Fraternalmente, João.

Ola' Shine, Joao e colegas da lista,acho que eu poderia melhorar a explicacao, mas vamos la' assim mesmo...Sempre podemos dividir os competidores da seguinte forma:Coloque o maior clique na sala "A" e todos os outros na sala "B".Se na sala "B" tambem houver um clique com o tamanho da sala "A", a divisao esta' completa. Se nao, execute a etapa X.Etapa X :Passe um competidor da sala "A" para a sala "B".Dessa forma, o clique em "A" diminui de 1 unidade, alguns cliques em "B" crescem de 1 unidade, e outros cliques em "B" nao se alteram.Entao:- Se o(s) maior(es) clique(s) em "B" ainda nao igualou o clique em "A", repita a etapa "X".- Se o(s) maior(es) clique(s) em "B" igualou o clique em "A", a divisao esta' completa.- E se o(s) maior(es) clique(s) em "B" ultrapassou o clique em "A" ?Bem, em cada um desses cliques (o clique formado pelos migrados de "A" nao esta' entre estes cliques, pois o clique original em "A" era par), existe algum competidor que nao estava originalmente em "A" .Passe esse competidor para "A" (faca isso em todos os cliques de "B" que ultrapassaram o valor em "A").Agora a divisao esta' completa.OBS: Poderia acontecer de todos os jogadores transferidos para "A" formarem um clique independente, superior ao clique em "A" ?Nao, caso contrario eles ja' estariam formando um clique na sala "B" igual ao clique em "A", antes da ultima passagem de alguem de "A" para "B", e o processo ja' teria terminado.Note que o clique original em "A" e' par. Assim, todo o processo descrito termina no maximo quando metade dos competidores em "A" tiver sido transferida para "B".[]'sRogerio PonceCarlos Yuzo Shine [EMAIL PROTECTED] escreveu:3. Numa competição de matemática, alguns competidores são amigos. Amizade é sempre mútua. Chame um grupo de competidores de clique se quaisquer dois entre eles são amigos. Em particular, qualquer grupo com menos de dois amigos é um clique. O número de membros de um clique é o seu tamanho.Dado que, nesta competição, o maior tamanho de um clique é par, prove que os competidores podem ser divididos em duas salas tais que o maior tamanho de um clique em uma sala é igual ao maior tamanho de um clique na outra sala.[]'sShineAlertas do Yahoo! Mail em seu celular. Saiba mais. Instruções para entrar na lista, sair da lista e usar a lista em
http://www.mat.puc-rio.br/~nicolau/olimp/obm-l.html
===

Re:Res:[obm-l] IMO 2007

2007-07-26 Por tôpico fernandobarcel
João,
clique é um grupo de competidores onde quaisquer dois entre eles são amigos.
Portanto, a competição pode não ser um clique.
Abraços,


-- Início da mensagem original ---
 
 Tentativa ao terceiro problema
 A própria competição (que encerra todos os competidores) é clique, 
 pois
 : 1) Há alguns competidores amigos; 2) A amizade é mútua, então, há pelo 
 menos 
 dois amigos na competição. 
 ...
 
 
 
[EMAIL PROTECTED] escreveu: -
 
  
  3. Numa competição de matemática, alguns competidores são amigos.
  Amizade é sempre mútua. Chame um grupo de competidores de clique se
  quaisquer dois entre eles são amigos. Em particular, qualquer grupo
  com menos de dois amigos é um clique. O número de membros de um
  clique é o seu tamanho.
  Dado que, nesta competição, o maior tamanho de um clique é par,
  prove que os competidores podem ser divididos em duas salas tais
  que o maior tamanho de um clique em uma sala é igual ao maior
  tamanho de um clique na outra sala.



=
Instruções para entrar na lista, sair da lista e usar a lista em
http://www.mat.puc-rio.br/~nicolau/olimp/obm-l.html
=


Res:[obm-l] IMO 2007

2007-07-26 Por tôpico JoaoCarlos_Junior
Tentativa ao terceiro problema	A própria competição (que encerra todos os competidores) é clique, pois: 1) Há alguns competidores amigos; 2) A amizade é mútua, então, há pelo menos dois amigos na competição. 	No conjunto clique particular não há amigos, haja vista que a amizade é mútua, e, assim, num conjunto que contém dois amigos como subconjunto, sempre esses amigos existirão. Clique particular também pode ser o conjunto vazio. Pelo primeiro parágrafo e a afirmação de que na competição o maior tamanho de um clique é par, então, a competição possui um número par de jogadores. Com as afirmações acima, provar que: o conjunto competição pode ser divido em duas partes tais que número de jogadores de uma dessas partes (que encerra dois amigos no mínimo) é igual ao número de jogadores da outra parte. Ora, isso já foi provado acima, do 1º parágrafo ao 3º, haja vista que chegamos à conclusão que na competição há número par de jogadores, com no mínimo dois amigos em toda a competição, e, portanto, uma metade que contém esses dois amigos pode ir a uma sala e a outra metade para outra, não importando saber se essa última metade contém amigos ou não, já que ela será sempre clique, pela definição particular ou genérica de clique.[EMAIL PROTECTED] escreveu: -Para: obm-l@mat.puc-rio.brDe: Carlos Yuzo Shine [EMAIL PROTECTED]Enviado por: [EMAIL PROTECTED]Data: 25/07/2007 12:08Assunto: [obm-l] IMO 2007Saiu agora o primeiro dia, no site do Mathlinks: http://www.mathlinks.ro/resources.php?c=1cid=16year=2007Traduzindo:1. São dados os números reais a_1, a_2, ..., a_n. Para cada i, 1 = i = n, definad_i = max{a_j, 1 = j = i} - min{a_j, i = j = n}.Seja d = max{d_i, 1 = i = n}.a) Prove que, para todos reais x_1 = x_2 = ... = x_n,max{ |x_i - a_i|, 1 = i = n} = d/2 (*)b) Mostre que existem reais x_1 = x_2 = ... = x_n tais que a igualdade em (*) ocorre.2. Considere cinco pontos A, B, C, D, E tais que ABCD é um paralelogramo e BCED é um quadrilátero cíclico. Seja r uma reta passando por A. Suponha que r corte o interior do segmento DC em F e a reta BC em G. Suponha também que EF = EG = EC. Prove que r é a bissetriz do ângulo DAB.3. Numa competição de matemática, alguns competidores são amigos. Amizade é sempre mútua. Chame um grupo de competidores de clique se quaisquer dois entre eles são amigos. Em particular, qualquer grupo com menos de dois amigos é um clique. O número de membros de um clique é o seu tamanho.Dado que, nesta competição, o maior tamanho de um clique é par, prove que os competidores podem ser divididos em duas salas tais que o maior tamanho de um clique em uma sala é igual ao maior tamanho de um clique na outra sala.[]'sShine   Get the free Yahoo! toolbar and rest assured with the added security of spyware protection.http://new.toolbar.yahoo.com/toolbar/features/norton/index.php=Instruções para entrar na lista, sair da lista e usar a lista emhttp://www.mat.puc-rio.br/~nicolau/olimp/obm-l.html=Instruções para entrar na lista, sair da lista e usar a lista em
http://www.mat.puc-rio.br/~nicolau/olimp/obm-l.html
===

[obm-l] IMO 2007

2007-07-25 Por tôpico Carlos Yuzo Shine
Saiu agora o primeiro dia, no site do Mathlinks: 
http://www.mathlinks.ro/resources.php?c=1cid=16year=2007

Traduzindo:

1. São dados os números reais a_1, a_2, ..., a_n. Para cada i, 1 = i = n, 
defina
d_i = max{a_j, 1 = j = i} - min{a_j, i = j = n}.

Seja d = max{d_i, 1 = i = n}.

a) Prove que, para todos reais x_1 = x_2 = ... = x_n,
max{ |x_i - a_i|, 1 = i = n} = d/2 (*)

b) Mostre que existem reais  x_1 = x_2 = ... = x_n tais que a igualdade em 
(*) ocorre.

2. Considere cinco pontos A, B, C, D, E tais que ABCD é um paralelogramo e BCED 
é um quadrilátero cíclico. Seja r uma reta passando por A. Suponha que r corte 
o interior do segmento DC em F e a reta BC em G. Suponha também que EF = EG = 
EC. Prove que r é a bissetriz do ângulo DAB.

3. Numa competição de matemática, alguns competidores são amigos. Amizade é 
sempre mútua. Chame um grupo de competidores de clique se quaisquer dois entre 
eles são amigos. Em particular, qualquer grupo com menos de dois amigos é um 
clique. O número de membros de um clique é o seu tamanho.

Dado que, nesta competição, o maior tamanho de um clique é par, prove que os 
competidores podem ser divididos em duas salas tais que o maior tamanho de um 
clique em uma sala é igual ao maior tamanho de um clique na outra sala.

[]'s
Shine


   

Get the free Yahoo! toolbar and rest assured with the added security of spyware 
protection.
http://new.toolbar.yahoo.com/toolbar/features/norton/index.php

=
Instruções para entrar na lista, sair da lista e usar a lista em
http://www.mat.puc-rio.br/~nicolau/olimp/obm-l.html
=


[obm-l] IMO

2006-12-28 Por tôpico Klaus Ferraz
(IMO-89)
 Mostre que, para cada natural n, existem n inteiros positivos consecutivos tais
que nenhum deles é um primo ou potência de primo.

(IMO) 
Mostre que existem n naturais consecutivos tais que nenhum deles possa ser
escrito como a soma de dois quadrados.

Grato.

__
Fale com seus amigos  de graça com o novo Yahoo! Messenger 
http://br.messenger.yahoo.com/ 

Re: [obm-l] IMO

2006-12-28 Por tôpico Carlos Yuzo Shine
Oi Klaus,

Esses dois problemas são bons exemplos de aplicações
do Teorema Chinês dos Restos: se k = 1 e m_1, m_2,
..., m_k são inteiros primos dois a dois (isto é, o
mdc entre quaisquer dois desses números é 1) então
existe x tal que x = a_1 (mód m_1), x = a_2 (mód m_2),
..., x = a_k (mód m_k), sendo a_1, a_2, ..., a_k
inteiros (assumo aqui que o leitor saiba o conceito de
congruência módulo m).

Para o problema da IMO 89, sendo x+1, x+2, ..., x+n os
n números, basta notar que, pelo Teorema Chinês dos
Restos, existe x tal que x = -1 (mód p_1p_2), x = -2
(mód p_3p_4), ..., x = -n (mód p_{2n-1}p_{2n}). Note
que cada um dos n números x+1, x+2, ..., x+n é
divisível por dois primos, então não pode ser primo ou
potência de primo.

Para o outro, vamos utilizar o fato de que se existe
um primo p da forma 4t+3 tal que p divide k e p^2 não
divide k então k não pode ser escrito como soma de
dois quadrados (utilize o Pequeno Teorema de Fermat
para provar isso). Sendo x+1, x+2, ..., x+n os
números, existe x tal que x+1 = p_1 (mód p_1^2), x+2 =
p_2 (mód p_2^2), ..., x+n = p_n (mód p_n^2), sendo
p_1, p_2, ..., p_n primos da forma 4t+3 (fica para o
leitor provar que existem infinitos desses primos).
Assim, cada um dos números é divisível por um primo da
forma 4t+3 mas não pelo sue quadrado, de modo que
nenhum dos n números é soma de dois quadrados.

[]'s
Shine

--- Klaus Ferraz [EMAIL PROTECTED] wrote:

 (IMO-89)
  Mostre que, para cada natural n, existem n inteiros
 positivos consecutivos tais
 que nenhum deles é um primo ou potência de primo.
 
 (IMO) 
 Mostre que existem n naturais consecutivos tais que
 nenhum deles possa ser
 escrito como a soma de dois quadrados.
 
 Grato.
 
 __
 Fale com seus amigos  de graça com o novo Yahoo!
 Messenger 
 http://br.messenger.yahoo.com/ 


__
Do You Yahoo!?
Tired of spam?  Yahoo! Mail has the best spam protection around 
http://mail.yahoo.com 
=
Instruções para entrar na lista, sair da lista e usar a lista em
http://www.mat.puc-rio.br/~nicolau/olimp/obm-l.html
=


Re: [obm-l] IMO

2006-12-28 Por tôpico Gabriel Ponce

Oi,
Eu respondi esta primeiro questão no mathlinks:
http://www.mathlinks.ro/Forum/viewtopic.php?t=113953 .
Tchau tchau


Em 28/12/06, Klaus Ferraz [EMAIL PROTECTED] escreveu:


 (IMO-89)

 Mostre que, para cada natural n, existem n inteiros positivos
consecutivos tais

que nenhum deles é um primo ou potência de primo.



(IMO)

Mostre que existem n naturais consecutivos tais que nenhum deles possa ser

escrito como a soma de dois quadrados.



Grato.

__
Fale com seus amigos de graça com o novo Yahoo! Messenger
http://br.messenger.yahoo.com/



[obm-l] IMO 2006 Eslovênia

2006-07-13 Por tôpico Marcio Cohen

   Prezados participantes da lista,

   A IMO 2006 já está disponível, inclusive com as soluções oficiais. Eu as 
coloquei em www.majorando.com , mas também é possível encontrá-las no site 
oficial dessa IMO.


   Esse site foi criado por mim e pelo Rodrigo Villard (ele já foi um 
participante ativo dessa lista).


   Nele você encontrará detalhes de um livro que acabamos de escrever e 
será lançado em agosto com tópicos teóricos e soluções das provas de 
matemática do IME dos últimos 15 anos.


   Encontrará também diversos artigos escritos por nós relacionados a 
olimpíadas de matemática, incluindo artigos de preparação para o vestibular 
do IME, olimpíadas de ensino médio (níveis intermediário e avançado) e 
olimpíadas universitárias (nível avançado).


Abraços,
Marcio Cohen


=
Instruções para entrar na lista, sair da lista e usar a lista em
http://www.mat.puc-rio.br/~nicolau/olimp/obm-l.html
=


[obm-l] IMO 2006 Eslov�nia

2006-07-13 Por tôpico Carlos Yuzo Shine
Eu acabei traduzindo os enunciados do segundo dia,
então aí vão eles:

E vamos torcer pelos nossos estudantes!

4. Encontre todos os pares (x,y) de inteiros tais que
1 + 2^x + 2^{2x+1} = y^2

5. Seja P(x) um polinômio de grau n  1 com
coeficientes inteiros e seja k um inteiro positivo.
Considere o polinômio Q(x) =
P(P(\ldots(P(P(x))\ldots)), em que P é aplicado k
vezes. Prove que existem no máximo n inteiros t tais
que Q(t) = t.

6. Associe a cada lado b de um polígono convexo P a
maior área de um triângulo que tem b como um de seus
lados e está contido em P. Prove que a soma das áreas
associadas aos lados de P é pelo menos o dobro da área
de P.

[]'s
Shine

__
Do You Yahoo!?
Tired of spam?  Yahoo! Mail has the best spam protection around 
http://mail.yahoo.com 
=
Instruções para entrar na lista, sair da lista e usar a lista em
http://www.mat.puc-rio.br/~nicolau/olimp/obm-l.html
=


[obm-l] IMO

2004-08-06 Por tôpico João Vitor
Qual foi a Equipe Brasileira da IMO este ano?

=
Instruções para entrar na lista, sair da lista e usar a lista em
http://www.mat.puc-rio.br/~nicolau/olimp/obm-l.html
=


Re: [obm-l] IMO

2004-08-06 Por tôpico Johann Peter Gustav Lejeune Dirichlet
Va ao site da OBM e veja! No link competiçoes, va em Olimpiada InternacionalJoão_Vitor [EMAIL PROTECTED] wrote:
Qual foi a Equipe Brasileira da IMO este ano?=Instruções para entrar na lista, sair da lista e usar a lista emhttp://www.mat.puc-rio.br/~nicolau/olimp/obm-l.html=
TRANSIRE SVVM PECTVS MVNDOQVE POTIRI CONGREGATI EX TOTO ORBE MATHEMATICI OB SCRIPTA INSIGNIA TRIBVERE Fields Medal(John Charles Fields)

N.F.C. (Ne Fronti Crede)
		Yahoo! Acesso Grátis - navegue de graça com conexão de qualidade!

Re: [obm-l] IMO 2004 - Problema 3 - Incompleto

2004-07-20 Por tôpico Carlos Yuzo Shine
Nesta IMO houve quatro Ouros 42: um do Canadá (note
que o Canadá empatou com o Brasil em pontos!!), um da
Hungria, e dois da Rússia. Nenhum é chinês ou
norte-americano.

Mas a delegação da China foi a única que obteve seis
medalhas de ouro este ano.

[]'s
Shine

--- [EMAIL PROTECTED] wrote:
 Falando em IMO sera que algum participante da China,
 U.S.A ou outro pais fez 
 42 pontos ? 
 
 
 
 
 Em uma mensagem de 19/7/2004 21:39:22 Hora padrão
 leste da Am. Sul, 
 [EMAIL PROTECTED] escreveu:
 
 
  
  Desculpem, mas este esboço que enviei está
 incompleto. Não li todas as 
  mensagens da lista, talvez alguém já tenha
 percebido, mas exatamente   onde 
  eu escrevo verifiquem isto, na pressa eu posso
 ter me enganado eu 
  realmente me enganei (a pressa não combina com
 problemas da IMO).
  
  Luciano.
  
  
  
 
 
 


__
Do You Yahoo!?
Tired of spam?  Yahoo! Mail has the best spam protection around 
http://mail.yahoo.com 
=
Instruções para entrar na lista, sair da lista e usar a lista em
http://www.mat.puc-rio.br/~nicolau/olimp/obm-l.html
=


[obm-l] IMO 2005, 2006, 2007, 2008 e 2009

2004-07-20 Por tôpico Carlos Yuzo Shine
Oi gente,

Só informando onde e quando serão as próximas IMOs:

1 a 12 de julho de 2005: Cancún, México (as provas
serão nos dias 6 e 7)
2006: Eslovênia
2007: Vietnam
2008: Espanha
2009: Alemanha

[]'s
Shine




__
Do you Yahoo!?
Vote for the stars of Yahoo!'s next ad campaign!
http://advision.webevents.yahoo.com/yahoo/votelifeengine/
=
Instruções para entrar na lista, sair da lista e usar a lista em
http://www.mat.puc-rio.br/~nicolau/olimp/obm-l.html
=


Re: [obm-l] IMO 2004 - Problema 3 - Incompleto

2004-07-19 Por tôpico Luciano Castro
Desculpem, mas este esboço que enviei está incompleto. Não li todas as 
mensagens da lista, talvez alguém já tenha percebido, mas exatamente   onde 
eu escrevo verifiquem isto, na pressa eu posso ter me enganado eu 
realmente me enganei (a pressa não combina com problemas da IMO).

Luciano.
At 19:01 13/07/04 -0300, you wrote:
Esboço de solução:
?xml:namespace prefix = o ns = urn:schemas-microsoft-com:office:office 
/Analisando as possibilidades para cobrir um canto do tabuleiro, nos 
convencemos de que a cobertura deve ser feita com retângulos 3x4 
(verifiquem isto, na pressa eu posso ter me enganado). Agora usamos o 
seguinte lema, cuja demonstração é fácil encontrar por aí:

Lema: Se um retângulo pode ser coberto com retangulinhos, cada um dos 
quais tem um lado com medida inteira, então o retângulo coberto tem um 
lado com medida inteira.

Suponha que temos um retângulo coberto com retângulos 3x4. Dividindo todos 
os lados por 3 (por 4) e aplicando o lema, descobrimos que um dos lados do 
retângulo original é múltiplo de 3 (de 4). Se o lado múltiplo de 3 é 
diferente do lado múltiplo de 4, acabou. Caso contrário, temos um lado 
múltiplo de 12 e precisamos determinar os possíveis valores do outro lado. 
Mas estes são precisamente os números inteiros positivos n que podem ser 
escritos da forma n = 3x + 4y, com x e y inteiros não negativos (aqui 
usamos o fato de o outro lado ser múltiplo de 12). Sabe-se que n pode ser 
qualquer inteiro maior ou igual a 12. Os outros valores determinam-se 
facilmente por inspeção  (desculpem-me por não finalizar, mas tenho pouco 
tempo).

- Mensagem Original 
De: [EMAIL PROTECTED]
Para: [EMAIL PROTECTED] [EMAIL PROTECTED]
Assunto: Re: [obm-l] IMO 2004 - Primeiro Dia
Data: 13/07/04 14:54
Ainda não pensei no problema 2, mas tenho a sensação de que nosso pessoal 
tem ótimas chances de fazer o 3. O problema 1 tenho certeza de que nossos 
6 alunos fizeram. Ainda não tive tempo de olhar o 2o dia.

Quando tiver mais tempo mando um esboço de solução para o 3, a menos que 
alguém o faça antes


Luciano.

- Mensagem Original 
De: [EMAIL PROTECTED]
Para: [EMAIL PROTECTED] [EMAIL PROTECTED]
Assunto: Re: [obm-l] IMO 2004 - Primeiro Dia
Data: 13/07/04 13:55



No proprio link ha uma discussao sobre pontos ... alguem já tem ideia dos
meninos??? O pessoal da Alemanha espera ouro com 33pts.

--
Le présent message ainsi que ses éventuelles pièces jointes est
exclusivement destiné au(x) destinataire(s), personnes physiques ou
morales, qursquo;il désigne.
Il constitue de ce fait une correspondance à caractère privé et peut
contenir des informations confidentielles.
Si ce message vous est parvenu par erreur, nous vous remercions drsquo;en 
aviser
immédiatement lrsquo;expéditeur par retour de courrier électronique puis 
de le
détruire, ainsi que ses éventuelles pièces jointes, sans en conserver de
copie.


This message, including any attachment, is intended for the use of the
individual or entity to which it is addressed.
It is therefore to be considered as a private correspondence which may
contain confidential information.
If you are not the intended recipient, please advise the sender immediately
by reply e.mail and delete this message and any attachment thereto without
retaining a copy.
--
=
Instruções para entrar na lista, sair da lista e usar a lista em
http://www.mat.puc-rio.br/~nicolau/olimp/obm-l.htmlhttp://www.mat.puc-rio.br/~nicolau/olimp/obm-l.html 

=


WNET - www.wnetrj.com.br
= 
Instruções para entrar na lista, sair da lista e usar a lista em 
http://www.mat.puc-rio.br/~nicolau/olimp/obm-l.html 
=



WNET - www.wnetrj.com.br
= 
Instruções para entrar na lista, sair da lista e usar a lista em 
http://www.mat.puc-rio.br/~nicolau/olimp/obm-l.html 
=

=
Instruções para entrar na lista, sair da lista e usar a lista em
http://www.mat.puc-rio.br/~nicolau/olimp/obm-l.html
=


Re: [obm-l] IMO 2004 - Problema 3 - Incompleto

2004-07-19 Por tôpico Faelccmm
Falando em IMO sera que algum participante da China, U.S.A ou outro pais fez 42 pontos ? 




Em uma mensagem de 19/7/2004 21:39:22 Hora padrão leste da Am. Sul, [EMAIL PROTECTED] escreveu:



Desculpem, mas este esboço que enviei está incompleto. Não li todas as 
mensagens da lista, talvez alguém já tenha percebido, mas exatamente onde 
eu escrevo "verifiquem isto, na pressa eu posso ter me enganado" eu 
realmente me enganei (a pressa não combina com problemas da IMO).

Luciano.







[obm-l] IMO - 2o DIA

2004-07-14 Por tôpico marciocohen
 As 
questoes do 2o dia de prova ja estao disponiveis. Como no 1o, eu tentei 
fazer serio essas questoes ontem, mas dessa vez eu nao aguentei (dois dias 
normais de trabalho seguidos de 4hs de problemas cansam :) ) e dormi antes.. 
Nao faltava mto tempo porem. Consegui fazer o 4 e acho que o 5 saiu, mas foi 
uma conta muito grande que precisa ser conferida. O 6 pra variar eu 
nao tentei :) Como diz o Luciano (ele diz o contrario na verdade!), 3 e 6 
assustam. :) 
 Eu estou escrevendo esses enunciados de cabeca (eu pensei neles 
ontem) mas acho que estao todos certos. Vou mandar em outro email as 
solucoes dos 4,5.. 
 Abraços, 
 Marcio 
 
4. Sejam t1, t2, ..., tn numeros reais positivos tais que 
(t1+t2+...+tn)(1/t1 + 1/t2 + 1/tn)  n^2 + 1. Mostre que todas as triplas 
da forma (ti, tj, tk) formam lados de triangulo. 
 
5. Seja ABCD um quadrilátero convexo tal que a diagonal BD nao eh 
bissetriz de ABC nem de ADC. Seja P no interior de ABCD tal que PBC 
= DBA e PDC = ADB. Mostre que AP=CP se e somente se 
o quadrilátero ABCD eh inscritivel.  
 
6. Um numero n eh dito alternado se em sua representacao decimal 
digitos consecutivos tem paridades distintas. Determine todos os naturais n 
que admitem um multiplo alternado. 
 

___
Super iG - Internet em Alta Velocidade - http://www.superig.com.br/

=
Instruções para entrar na lista, sair da lista e usar a lista em
http://www.mat.puc-rio.br/~nicolau/olimp/obm-l.html
=


[obm-l] IMO 2004 Problemas 4 e 5

2004-07-14 Por tôpico marciocohen
Problema 4: 
Mostre que se t1,t2,...,tn sao reais positivos e (t1+...+tn)(1/t1 + ... 
+ 1/tn)  n^2 + 1 entao (ti,tk,tk) sempre podem formar um  
triangulo. 
Solucao: Vamos mostrar que se t1,t2,t3,...,tn sao reais 
positivos tais que (spg) t1t2+t3,  
entao(t1+t2+...+tn)(1/t1+ ... + 1/tn) = n^2 + 1. (o problema 
pede a contrapositiva disso). 
 O caso geral reduz diretamente ao caso n=3:  Sejam 
a,b,c,x positivos tq a = b+c+x. Entao:(a+b+c)(1/a + 1/b + 1/c) = 
(b+c+b+c+x)( 1/(b+c+x) + 1/b + 1/c) =(b+c)/(b+c+x) + 1 + 2[(b+c)/b + 
(b+c)/c] + x(b+c)/bc  Como (b+c)/b + (b+c)/c = 2 + c/b+b/c = 
4, LE = 10 se (b+c)/(b+c+x) + x(b+c)/bc = 1 sss bc(b+c) + 
x(b+c)(b+c+x) = bc(b+c+x)sss x(b+c)(b+c+x) = xbc que eh verdade 
pq (b+c)(b+c+x) = (b+c)^2 = 4bc. 
 Para n3, eh uma inducao boba. Suponha valido para 
n. Suponha ab+c. Ponha A=a+b+c..+t_n-1, B = 1/a + 1/b + ... + 
1/t_n-1 Entao, (a+b+c+t4+...+tn)(1/a+1/b+1/c+...+1/tn) = 
(A+tn)(B+1/tn) = AB + A/tn + tnB + 1. Agora note que A/tn + tnB 
contem n-1 termos da forma ti/tn + tn/ti, e portanto essa  
expressao eh = 2(n-1).  Por hipotese de inducao, AB = 
(n-1)^2 + 1.  Logo, LE = (n-1)^2 + 1 +2(n-1) + 1 = n^2 + 
1.*** 
 
 
 
 
Problema 5: Quadrilatero ABCD convexo, P no interior satisfaz 
PBC=DBA e  
BDA=PDC. Mostre que ABCD eh inscritivel sss 
AP=CP. 
Solução:Eu fiz tudo por complexos. A ida eh bem direta, mas a 
volta ficou grande (nao tao  
grande assim comparados a alguns da aula de complexos da preparacao pra 
imo). 
= Suponha ABCD ciclico. Escolha os eixos tq 
aa'=bb'=cc'=dd'=1.PBC=DBA = 
[(p-b)/(c-b)]/[(a-b)/(d-b)] eh real =  
(p-b)(d-b)/[(c-b)(a-b)] = (p'-b')(d'-b')/[(c'-b')(a'-b')].Usando que 
c'-b'=1/c - 1/b =-(c-b)/bc, (c-b)/(c'-b')=-bc isso vira: 
bd(p-b) = -cbab(p'-1/b), ou seja: d(p-b) = 
-ac(bp'-1)Trocando b por d, a outra condicao portanto 
eh: b(p-d) = -ac(dp'-1)Subtraindo as equacoes: p(d-b) = 
-acp'(b-d), ou p + acp' = 0, ou seja, p esta na  
mediatriz da corda AC, e portanto AP=AC.  
= Eu ate tentei dar um argumento geometrico baseado na ida, mas fui 
incapaz e resolvi  
fazer logo a conta.. Eu acho q a unica pessoa que vai ler isso tentando 
entender serah  
eu mesmo, daqui a uns 2 anos (como fiz recentemente p/ dar uma aula), 
mas td bem. :) Sem papel e caneta, nem pense em ler 
... Ponha a=-1, c=1, p no eixo vertical (aqui usei que 
AP=CP).  PBC=DBA = 
[(1-b)/(p-b)]/[(d-b)/(-1-b)] real, donde (como 
p'=-p): (b^2-1)/[(p-b)(b-d)] = 
(1-b'^2)/[(p+b')(b'-d')](i) Analogamente, 
(d^2-1)/[(p-d)(d-b)] = (1-d'^2)/[(p+d')(d'-b')] (ii) 
Provar que b,-1,d,1 estao no mesmo circulo eh mostrar que 
 [(b+1)(d-1)]/[(b-1)(d+1)] eh real. (iii).A estratégia 
para mostrar q (i) e (ii) implicam (iii) seria: Eliminar p 
de (i),(ii) Expandir (iii) . 
Comparar/Conferir/Comparar.  Aqui eu cheguei a fazer 
bastante conta mas cansei. Acho que dá para continuar e terminar (afinal, 
sao soh polinomios e fatoracoes), um dia eu retomo isso (qdo for dar outra 
aula de complexos provavelmente :).  Vou esperar 
um pouco antes de ver solucao pra esse pra ver se depois pego essas contas e 
termino (a estimativa eh uma conta com uns 64 termos de cada lado no pior 
dos casos. daí tentar fatorar um (b-d) e comparar). 
 
 Abraços, Marcio 

___
Super iG - Internet em Alta Velocidade - http://www.superig.com.br/

=
Instruções para entrar na lista, sair da lista e usar a lista em
http://www.mat.puc-rio.br/~nicolau/olimp/obm-l.html
=


Re: [obm-l] IMO 2004 Problemas 5 - closing

2004-07-14 Por tôpico marciocohen
 
Esquecam a conta para a volta do problema. Os argumentos geometricos 
tradicionais usando a idafuncionam sim e a solucao fica bem mais 
simples.. (embora na primeira tentativa eu tenha desistido e ido pra 
conta).Quem vai ser boa alma que vai postar aqui o desempenho do 
Brasil na prova? Estou torcendo pra ver que todos fizeram a 5 por complexos 
:)  
 Suponha AP=CP e olhe para o circuncirculo i de ABC. Extenda 
BD até D' em i. O quadrilatero ABCD' da origem a um novo ponto P' na reta BP 
satisfazendo as condicoes de angulo doenunciado, e pela ida, P'A=P'C. 
Logo P' esta na mediatriz de AC e em BP, donde P'=P e entao D=D' esta em 
i. 
Obs: Pensei bastantena 6mas travei. Aceito sugestoes! 
Conjecturo que todo n funciona. Isso deve valer 0 ponto :) 
*** 
 
 
 Suponha ABCD ciclico. Escolha os eixos tq aa'=bb'=cc'=dd'=1. 
 = [(p-b)/(c-b)]/[(a-b)/(d-b)] eh real 
=(p-b)(d-b)/[(c-b)(a-b)] = (p'-b')(d'-b')/[(c'-b')(a'-b')]. 
Usando que c'-b'=1/c - 1/b =-(c-b)/bc, (c-b)/(c'-b')=-bc isso 
vira: 
 
bd(p-b) = -cbab(p'-1/b), ou seja: 
 d(p-b) = -ac(bp'-1) 
Trocando b por d, a outra condicao portanto eh: 
 b(p-d) = -ac(dp'-1) 
Subtraindo as equacoes: p(d-b) = -acp'(b-d), ou p + acp' = 0, ou 
seja, p esta na 
 mediatriz da corda AC, e portanto AP=AC. 
 

___
Super iG - Internet em Alta Velocidade - http://www.superig.com.br/

=
Instruções para entrar na lista, sair da lista e usar a lista em
http://www.mat.puc-rio.br/~nicolau/olimp/obm-l.html
=


Re: [obm-l] IMO - 2o DIA

2004-07-14 Por tôpico Domingos Jr.
4. Sejam t1, t2, ..., tn numeros reais positivos tais  que 
(t1+t2+...+tn)(1/t1 + 1/t2 + 1/tn)  n^2 + 1. Mostre que todas as 
triplas da forma (ti, tj, tk) formam lados de triangulo.
Não quis ver sua resposta ainda (espero que não seja nada muito parecido 
ao que você já mandou), mas parece que dá pra provar algo mais forte do 
que é pedido...

Fixe um S  0. Se t_1 + ... + t_n = S, qual o menor valor possível para
(t_1 + ... + t_n)(1/t_1 + ... + 1/t_n) ?
Podemos minimizar 1/t_1 + ... + 1/t_n sujeito a t_1 + ... + t_n = S e 
t_i  0.
Por Lagrange, temos que o mínimo ocorre quando t_1 = t_2 = ... = t_n = S/n.
Então esse mínimo é 1/(S/n) + ... + 1/(S/n) = n^2/S.
t_1 + ... + t_n = S e o menor valor possível para
1/t_1 + ... + 1/t_n = n^2/S, logo o menor valor possível para
(t_1 + ... + t_n)(1/t_1 + ... + 1/t_n) é n^2 e isso ocorre somente 
quando t_1 = ... = t_n... como estamos limitados superiormente em n^2+1 
é de se esperar que o desvio padrão dos t_i's seja muito pequeno (o que 
é uma condição mais forte do que a enunciada, daria pra falar que os
triângulos são quase equiláteros).

O caso n = 3 sai sem problemas, mas não vou colocar minha aqui.
[ ]'s
=
Instruções para entrar na lista, sair da lista e usar a lista em
http://www.mat.puc-rio.br/~nicolau/olimp/obm-l.html
=


[obm-l] IMO 2004 -Primeiro e Segundo dia

2004-07-13 Por tôpico Paulo Rodrigues
A prova está em

http://www.teorema.mat.br/noticias.html

Paulo

=
Instruções para entrar na lista, sair da lista e usar a lista em
http://www.mat.puc-rio.br/~nicolau/olimp/obm-l.html
=


Re: [obm-l] IMO 2004 - Primeiro Dia

2004-07-13 Por tôpico Domingos Jr.
A prova do primeiro dia da IMO (em inglês), está em
 
http://www.teorema.mat.br/imo20041.pdf
 
Paulo
http://www.teorema

Gostei do segundo... Eu conjecturo que a resposta é f(x) = C.x^2, para 
qualquer constante real C.

Algumas idéias:
Se a = b = c = 0, temos
3f(0) = 2f(0) = f(0) = 0
Se b = c = 0, a fica livre (pois ab + bc + ac = 0 independente do valor 
de a).
f(a-b) + f(b-c) + f(c-a) = f(a) + f(0) + f(-a) = f(a) + f(-a)
e f(a+b+c) = f(a), logo
f(a) + f(-a) = 2f(a) = f(a) = f(-a) para todo a real = f é função par.

Seja u um real, note que se (a, b, c) é uma tripla satisfazendo ab + bc 
+ ac, temos que u(a, b, c) = (ua, ub, uc) também satisfaz (ua)(ub) + 
(ub)(uc) + (ua)(uc) = u^2(ab + bc + ac) = 0.

Sendo assim,
f(u(a-b)) + f(u(b-c)) + f(u(c-a)) = 2f(u(a+b+c)) para todo u real.
Podemos então encarar a igualdade acima como uma igualdade de duas 
funções de u, e podemos aplicar derivadas a ambos os lados já que f é de 
classe C^oo.

Se f é um polinômio de grau 2n, a 2n-ésima derivada de f é constante (o 
coeficiente líder do polinômio).

Veja que d^k [f(u(a-b))]/du = (a-b)^k * f^(k)(u(a-b)) --
onde f^(k)(x0) é a k-ésima derivada de f aplicada em x0.
Como f^(2n)(x) = alpha (constante), devemos ter
(a-b)^2n + (b-c)^2n + (c-a)^2n = 2(a+b+c)^2n.
Agora vem a conjectura: parece que o lado direito cresce mais (com 
relação a n) que o lado esquerdo... mas isso é palpite, precisa fazer 
conta pra mostrar algo do tipo...

[ ]'s
=
Instruções para entrar na lista, sair da lista e usar a lista em
http://www.mat.puc-rio.br/~nicolau/olimp/obm-l.html
=


Re: [obm-l] IMO 2004 - Primeiro Dia

2004-07-13 Por tôpico Paulo Rodrigues
x^4 também funciona.

Paulo
- Original Message -
From: Domingos Jr. [EMAIL PROTECTED]
To: [EMAIL PROTECTED]
Sent: Tuesday, July 13, 2004 3:25 PM
Subject: Re: [obm-l] IMO 2004 - Primeiro Dia


 A prova do primeiro dia da IMO (em inglês), está em

 http://www.teorema.mat.br/imo20041.pdf

 Paulo
 http://www.teorema


Gostei do segundo... Eu conjecturo que a resposta é f(x) = C.x^2, para
qualquer constante real C.

Algumas idéias:

Se a = b = c = 0, temos
3f(0) = 2f(0) = f(0) = 0

Se b = c = 0, a fica livre (pois ab + bc + ac = 0 independente do valor
de a).
f(a-b) + f(b-c) + f(c-a) = f(a) + f(0) + f(-a) = f(a) + f(-a)
e f(a+b+c) = f(a), logo
f(a) + f(-a) = 2f(a) = f(a) = f(-a) para todo a real = f é função par.

Seja u um real, note que se (a, b, c) é uma tripla satisfazendo ab + bc
+ ac, temos que u(a, b, c) = (ua, ub, uc) também satisfaz (ua)(ub) +
(ub)(uc) + (ua)(uc) = u^2(ab + bc + ac) = 0.

Sendo assim,
f(u(a-b)) + f(u(b-c)) + f(u(c-a)) = 2f(u(a+b+c)) para todo u real.
Podemos então encarar a igualdade acima como uma igualdade de duas
funções de u, e podemos aplicar derivadas a ambos os lados já que f é de
classe C^oo.

Se f é um polinômio de grau 2n, a 2n-ésima derivada de f é constante (o
coeficiente líder do polinômio).

Veja que d^k [f(u(a-b))]/du = (a-b)^k * f^(k)(u(a-b)) --
onde f^(k)(x0) é a k-ésima derivada de f aplicada em x0.
Como f^(2n)(x) = alpha (constante), devemos ter
(a-b)^2n + (b-c)^2n + (c-a)^2n = 2(a+b+c)^2n.

Agora vem a conjectura: parece que o lado direito cresce mais (com
relação a n) que o lado esquerdo... mas isso é palpite, precisa fazer
conta pra mostrar algo do tipo...

[ ]'s
=
Instruções para entrar na lista, sair da lista e usar a lista em
http://www.mat.puc-rio.br/~nicolau/olimp/obm-l.html
=

=
Instruções para entrar na lista, sair da lista e usar a lista em
http://www.mat.puc-rio.br/~nicolau/olimp/obm-l.html
=


Re: [obm-l] IMO 2004 - Primeiro Dia

2004-07-13 Por tôpico marciocohen
 Eu havia mandado a solução dos dois primeiros 
problemas, bem como os enunciados dos tres primeiros (o 3 eu nao consegui 
fazer) para a obm-l, mas o email voltou nao sei pq (tinha um arquivo de 
miseros 2kb). Vou reenviar aqui o email: 
 A propósito, sua conjectura eh "quase" verdadeira, e tmb 
foi a minha conjectura durante boa parte da solução! 
" Oi gente, sairam as questoes do 1o dia 
de prova. Elas estao disponiveis em www.mathlinks.ro, mas voce 
deve tomar o cuidado de clicar soh no primeiro link para nao ver as 
solucoes. Meus comentarios seguem abaixo pra quem quiser ler. (Nao leia se 
vc ainda quer pensar!!) 
Problema 1: Seja ABC um triângulo acutângulo, 
AB != AC. O circulo de diâmetro BC intersecta AB e AC em M e N 
respectivamente. Seja O medio de BC. As bissetrizes de BOC e MON se 
encontram em R. Mostre que os circulos circunscritos aos triangulos BMR e 
CNR se intersectam num ponto de BC.  
 
Problema 2: Encontre todos os polinomios f com 
coeficientes reais tais que, para todos a,b,c com ab+ac+bc=0 se tem 
f(a-b)+f(b-c)+f(c-a)=2f(a+b+c). 
 
Problema 3:Defina um anzol como sendo uma 
figura formada a partir de 6 quadrados unitarios como mostrado na 
figuraem anexo,ou qualquer das figuras obtendo por uma rotacao 
desta. Determine todos os retangulos mxn que podem ser cobertos com anzois 
de modo que: 
 (a) O retangulo eh coberto 
sem buracos e sem sobreposicoes e 
 (b) Nenhuma parte do anzol cai fora do retangulo. 
 
 
 
 
 
 
 
**COMENTÁRIOS** 
 Eu consegui arranjar 4hs do 
meu dia pra pensar nas questoes.. Perdi muito tempo tentando fazer conta na 
1 (q eh bem facil, mas eu soh notei isso depois que desisti da conta e 
transportei angulo)e acabei nao pensando mto na 3 (q eu provavelmente 
nao faria mesmo :) ). Eu consegui fazer os problemas 1 e 2, . O dois nao eh 
tao direto qto o 1 (pelo menos nao foi na minha solucao).Espero nao 
ter errado nada.. Pensem tmb, corrijam-me se for o casoe mandem pra 
lista alguma coisa. Amanha espero conseguir separar um tempo pra pensar nos 
outros 3 (q ainda nao estao disponiveis no site).  
 
 Minhas solucoes por enquanto 
(Nao coloquei aqui passo a passo pq eh chato digitar. se alguem se 
manifestar ou se eu tiver tempo livre depois eu posso colocar com mais 
detalhes). 
 
SOLUCAO DO 1*** 
1. a. Incialmente mostre que MR = NR 
(congruencia de triangulos OMR e ONR).  
 b. Note que a condicao do 
problema eh equivalente a mostrar que BMR+RNC = 180.  
 c. Aplique lei dos senos em 
AMR e ANR p/ concluir que sen(BMR)=sen(RNC). 
 d. Usando a congruencia 
(a),veja que BMR=RNC = ABC isosceles, logo 
BMR+RNC=180. 
 
 
 
 
 
 
 
 
 
 
 
SOLUCAO DO 2 
2. Note primeiro que f eh par (a=b=c da f(0) = 
0 e dai ponha (a,b,c) = (-2c,-2c,c) por exemplo). 
 Ponha (a,b,c) = (-x, 1+x, 
x+x^2)e note que ab+ac+bc = 0para todo x real(a idéia aqui 
foi arriscar um pouco fazendo a=-1 e depois limpar 
denominadores). 
Substituindo 
a,b,cna eq.funcional: f(2x+1) + f(x^2-1) + f(2x+x^2) = 
2*f(x^2+x+1) p/ todo x real (*) 
 Como f eh um 
polinomiopar, escreva f(x) = a*x^n + b*x^(n-2) + ...graus 
menores. 
 Compare agora os 
coeficientes de x^(2n-2) em cada lado de (*). Para 2n-2n (i.e, n2), 
o termo f(2x+1) nao influencia e nessa comparacao(usando binomio de 
newton)soh os coeficientes com "a" interferem, dando algo como 
a*[-n + 4Binomial(n,2)] = 2a*[Binomial(n,2) + n], donde n = 0 ou n = 4. 
 
 Logo, estamos restritos a 
f(x) = p*x^4 + q*x^2 .Aqui, para a conta nao ficar grande, note que 
f(x)=qx^2 claramente satisfaz as condições do problema para qualquer q. Mais 
ainda, pondo f(x) = g(x) + q*x^2, note que f satisfazo 
problemasse g satisfaz. Logo, basta mostrar que g(x)=p*x^4 tmb 
satisfaz a igualdade.  Isso demorou um pouco pra mim (eu 
inclusive cheguei aachar q nao funcionava): 
 Mas funciona.Meu modo 
de ver isso foi o seguinte: Pondo f(x) = p*x^4 ea condicao em (a,b,c) 
voce cai num problema todo homogeneo. Logo, vc pode fazer spg a = -1, donde 
b = c/(c-1). Escrevendo c=1+x e multiplicando tudo por x, vc conclui q soh 
precisa mostrar p/ (a,b,c)=(-x,1+x,x+x^2), ou seja, eh soh testar em (*). Ai 
eu comparei no braço os 2 polinomios de grau 8.. Talvez vcs achem mais facil 
substituir direto na eq. funcional. 
 Conclusao: Os unicos 
polinomios que satisfazem o enunciado sao aqueles da forma f(x) = p*x^4 + 
q*x^2, com p,q reais. 
 
  
  
 Abraços aos que leram até 
aqui!  
 Marcio!" 
 
 
Em 13 Jul 2004, [EMAIL PROTECTED] escreveu:  
 A prova do primeiro dia da IMO (em inglês), está em 
 
 http://www.teorema.mat.br/imo20041.pdf 
 
 Paulo 
 
 
Gostei do segundo... Eu conjecturo que a resposta é f(x) = 
C.x^2, para 
qualquer constante real C. 
 
Algumas idéias: 
 
Se a = b = c = 0, temos 
3f(0) = 2f(0) = f(0) = 0 
 
Se b = c = 0, a fica livre (pois ab + bc + ac = 0 
independente do valor 
de a). 
f(a-b) + f(b-c) + f(c-a) = f(a) + f(0) + f(-a) = f(a) + f(-a) 
e f(a+b+c) = f(a), logo 
f(a) + f(-a) = 2f(a) = f(a) = f(-a) para todo a real 

Re: [obm-l] IMO 2004 - Primeiro Dia

2004-07-13 Por tôpico Paulo-Andre . Melo




No proprio link ha uma discussao sobre pontos ... alguem j tem ideia dos
meninos??? O pessoal da Alemanha espera ouro com 33pts.


--
Le prsent message ainsi que ses ventuelles pices jointes est
exclusivement destin au(x) destinataire(s), personnes physiques ou
morales, quil dsigne.
Il constitue de ce fait une correspondance  caractre priv et peut
contenir des informations confidentielles.
Si ce message vous est parvenu par erreur, nous vous remercions den aviser
immdiatement lexpditeur par retour de courrier lectronique puis de le
dtruire, ainsi que ses ventuelles pices jointes, sans en conserver de
copie.


This message, including any attachment, is intended for the use of the
individual or entity to which it is addressed.
It is therefore to be considered as a private correspondence which may
contain confidential information.
If you are not the intended recipient, please advise the sender immediately
by reply e.mail and delete this message and any attachment thereto without
retaining a copy.
--

=
Instrues para entrar na lista, sair da lista e usar a lista em
http://www.mat.puc-rio.br/~nicolau/olimp/obm-l.html
=


Re: [obm-l] IMO 2004 - Primeiro Dia

2004-07-13 Por tôpico Lucianocastro
Ainda não pensei no problema 2, mas tenho a sensação de que nosso pessoal
tem ótimas chances de fazer o 3. O problema 1 tenho certeza de que nossos 6
alunos fizeram. Ainda não tive tempo de olhar o 2o dia.Quando tiver
mais tempo mando um esboço de solução para o 3, a menos que alguém o faça
antesLuciano.

- Mensagem Original De:
[EMAIL PROTECTED]Para: "[EMAIL PROTECTED]"
[EMAIL PROTECTED]Assunto: Re: [obm-l] IMO 2004 - Primeiro
DiaData: 13/07/04 13:55No proprio link ha uma discussao sobre pontos ...
alguem já tem ideia dosmeninos??? O pessoal da Alemanha espera ouro com
33pts.--Le
présent message ainsi que ses éventuelles pièces jointes
estexclusivement destiné au(x) destinataire(s), personnes physiques
oumorales, qursquo;il désigne.Il constitue de ce fait une
correspondance à caractère privé et peutcontenir des informations
confidentielles.Si ce message vous est parvenu par erreur, nous vous
remercions drsquo;en aviserimmédiatement lrsquo;expéditeur par
retour de courrier électronique puis de ledétruire, ainsi que ses
éventuelles pièces jointes, sans en conserver decopie.This
message, including any attachment, is intended for the use of
theindividual or entity to which it is addressed.It is therefore to
be considered as a private correspondence which maycontain confidential
information.If you are not the intended recipient, please advise the
sender immediatelyby reply e.mail and delete this message and any
attachment thereto withoutretaining a
copy.--==Instruções
para entrar na lista, sair da lista e usar a lista emhttp://www.mat.puc-rio.br/~nicolau/olimp/obm-l.html=


WNET - www.wnetrj.com.br


=
Instruções para entrar na lista, sair da lista e usar a lista em
http://www.mat.puc-rio.br/~nicolau/olimp/obm-l.html
=


[obm-l] IMO 2004 - Problema 3

2004-07-13 Por tôpico Lucianocastro
Esboço de solução:

Analisando as possibilidades para cobrir um canto do tabuleiro, nos
convencemos de que a cobertura deve ser feita com retângulos 3x4 (verifiquem
isto, na pressa eu posso ter me enganado). Agora usamos o seguinte lema,
cuja demonstração é fácil encontrar por aí:

Lema: Se um retângulo pode ser coberto com retangulinhos, cada um dos
quais tem um lado com medida inteira, então o retângulo coberto tem um lado
com medida inteira.

Suponha que temos um retângulo coberto com retângulos 3x4. Dividindo
todos os lados por 3 (por 4) e aplicando o lema, descobrimos que um dos
lados do retângulo original é múltiplo de 3 (de 4). Se o lado múltiplo de 3
é diferente do lado múltiplo de 4, acabou. Caso contrário, temos um lado
múltiplo de 12 e precisamos determinar os possíveis valores do outro lado.
Mas estes são precisamente os números inteiros positivos n que podem ser
escritos da forma n = 3x + 4y, com x e y inteiros não negativos (aqui usamos
o fato de o outro lado ser múltiplo de 12). Sabe-se que n pode ser qualquer
inteiro maior ou igual a 12. Os outros valores determinam-se facilmente por
inspeção (desculpem-me por não
finalizar, mas tenho pouco tempo).

- Mensagem Original De:
[EMAIL PROTECTED]Para: "[EMAIL PROTECTED]"
[EMAIL PROTECTED]Assunto: Re: [obm-l] IMO 2004 - Primeiro
DiaData: 13/07/04 14:54Ainda não pensei no problema 2, mas tenho
a sensação de que nosso pessoal tem ótimas chances de fazer o 3. O problema
1 tenho certeza de que nossos 6 alunos fizeram. Ainda não tive tempo de
olhar o 2o dia.Quando tiver mais tempo mando um esboço de solução
para o 3, a menos que alguém o faça
antesLuciano.

- Mensagem Original De:
[EMAIL PROTECTED]Para: "[EMAIL PROTECTED]"
[EMAIL PROTECTED]Assunto: Re: [obm-l] IMO 2004 - Primeiro
DiaData: 13/07/04 13:55No proprio link ha uma discussao sobre pontos ...
alguem já tem ideia dosmeninos??? O pessoal da Alemanha espera ouro com
33pts.--Le
présent message ainsi que ses éventuelles pièces jointes
estexclusivement destiné au(x) destinataire(s), personnes physiques
oumorales, qursquo;il désigne.Il constitue de ce fait une
correspondance à caractère privé et peutcontenir des informations
confidentielles.Si ce message vous est parvenu par erreur, nous vous
remercions drsquo;en aviserimmédiatement lrsquo;expéditeur par
retour de courrier électronique puis de ledétruire, ainsi que ses
éventuelles pièces jointes, sans en conserver decopie.This
message, including any attachment, is intended for the use of
theindividual or entity to which it is addressed.It is therefore to
be considered as a private correspondence which maycontain confidential
information.If you are not the intended recipient, please advise the
sender immediatelyby reply e.mail and delete this message and any
attachment thereto withoutretaining a
copy.--==Instruções
para entrar na lista, sair da lista e usar a lista emhttp://www.mat.puc-rio.br/~nicolau/olimp/obm-l.html=WNET
-
www.wnetrj.com.br=
Instruções para entrar na lista, sair da lista e usar a lista em
http://www.mat.puc-rio.br/~nicolau/olimp/obm-l.html
=



WNET - www.wnetrj.com.br


=
Instruções para entrar na lista, sair da lista e usar a lista em
http://www.mat.puc-rio.br/~nicolau/olimp/obm-l.html
=


[obm-l] IMO 2004 - Primeiro Dia

2004-07-12 Por tôpico Paulo Rodrigues



A prova do primeiro dia da IMO (em inglês), está em

http://www.teorema.mat.br/imo20041.pdf

Paulo






[obm-l] imo

2003-08-17 Por tôpico gabriel



Ola pessoal,
queria saber se a imo shortlist 2002 ja foi 
liberada??se ja aonde posso encontrar???
Gabriel Guedes



Re: [obm-l] imo

2003-08-17 Por tôpico Eduardo Casagrande Stabel



Oi Gabriel.

No site do John Scholes (a enciclopédia olímpica da 
internet) tem a shortlist da IMO de 2002. Não sei se você quis digitar 2003. 
Bom, dê uma olhada

http://www.kalva.demon.co.uk

Abração!
Duda.

  - Original Message - 
  From: 
  gabriel 
  To: [EMAIL PROTECTED] 
  Sent: Sunday, August 17, 2003 5:46 
  PM
  Subject: [obm-l] imo
  
  Ola pessoal,
  queria saber se a imo shortlist 2002 ja foi 
  liberada??se ja aonde posso encontrar???
  Gabriel Guedes
  


[obm-l] IMO, curiosidade

2003-07-28 Por tôpico Nelson
Vi em algumas resoluções de vocêssobre questões olímpicas, a utilização de vários teoremas. Gostaria de saber qual é o nível exigido pelo, por exemplo, IMO, ou seja, é nivel de ensino médio?

Desde já, Grato,
NelsonConheça o novo Cadê? - Mais rápido, mais fácil e mais preciso.
Toda a web, 42 milhões de páginas brasileiras e nova busca por imagens!

Re: [obm-l] IMO, curiosidade

2003-07-28 Por tôpico Johann Peter Gustav Lejeune Dirichlet
Depende do que voce quer dizer com isso.O que
voce deve saber nao e um conhecimento
extremamente vasto,talvez uma ou mais coisinhas
sobre coisas que nao se ve em ensino medio,mas
nada que nao se possa aprender com paciencia e
dedicaçao.
Por exemplo,na IMO do Japao,o problema 1 se
baseava em induçao;o 2 em teoria dos numeros,e
talvez equaçoes de segundo grau;o 3 em vetores e
geometria,talvez um pouco de trigonometria;o 4
era de trigonometria;o 5 de desigualdades
elementares,como Chebyshev e Cauchy-Buniakowski;o
6 era uma aplicaçao desesperada de ordens,raizes
primitivas e Euler-Fermat.
Tudo isso(exceto o problema 3)pode ser encontrado
nas Eureka!s.
Espero ter ajudado.
 --- Nelson [EMAIL PROTECTED]
escreveu:  Vi em algumas resoluções de vocês
sobre
 questões olímpicas, a utilização de vários
 teoremas. Gostaria de saber qual é o nível
 exigido pelo, por exemplo, IMO, ou seja, é
 nivel de ensino médio?
  
 Desde já, Grato,
 Nelson
 
 
 
 -
 Conheça o novo Cadê? - Mais rápido, mais fácil
 e mais preciso.
 Toda a web, 42 milhões de páginas brasileiras e
 nova busca por imagens! 

___
Conheça o novo Cadê? - Mais rápido, mais fácil e mais preciso.
Toda a web, 42 milhões de páginas brasileiras e nova busca por imagens!
http://www.cade.com.br
=
Instruções para entrar na lista, sair da lista e usar a lista em
http://www.mat.puc-rio.br/~nicolau/olimp/obm-l.html
=


Re: [obm-l] Re: [obm-l] IMO - P1

2003-07-21 Por tôpico Johann Peter Gustav Lejeune Dirichlet
Quem quer generaliuzar???


 --- Marcio Afonso A. Cohen
[EMAIL PROTECTED] escreveu:  É
verdade! Valeu!
 Marcio
 
 - Original Message -
 From: [EMAIL PROTECTED]
 To: [EMAIL PROTECTED]
 Sent: Saturday, July 19, 2003 4:49 PM
 Subject: [obm-l] Re: [obm-l] IMO - P1
 
 
 
  Oi Marcio,
   Soh hj eu li seu email, depois que eu tbm
 consegui fazer a questão.
   Tem apenas um detalhe que vc não observou:
 os t_i´s devem ser distintos,
  pq senão os dois conjuntos seriam iguais.
   Seguindo a sua notação, sendo D_i=(D+
 t_i)U(t_i- D), temos |D_i|=
 2.5050.
  O t_(i+1) deve ser escolhido em
 T = S\(S_1 U...U S_i U {t_1,
 t_2,...,t_i})
Olha como o problema é impressionante: para
 garantir que t_100 pode ser
  escolhido, devemos ter T não-vazio. Ora,
  |S_1 U...U S_99 U {t_1, t_2,...,t_99}|=
 |S_1|+...+ |S_99|+ 99=
99.2.5050+ 99= 00+ 99= 99 
 100  ()
 Os números foram muitos bem escolhidos, e
 o problema ainda não perdeu
  a elegância com números feios! NOvamente,
 parabéns Gugu.
Ateh mais,
 

=
 Instruções para entrar na lista, sair da lista
 e usar a lista em

http://www.mat.puc-rio.br/~nicolau/olimp/obm-l.html

= 

___
Yahoo! Mail
Mais espaço, mais segurança e gratuito: caixa postal de 6MB, antivírus, proteção 
contra spam.
http://br.mail.yahoo.com/
=
Instruções para entrar na lista, sair da lista e usar a lista em
http://www.mat.puc-rio.br/~nicolau/olimp/obm-l.html
=


Re: [obm-l] IMO - Curiosidades.

2003-07-21 Por tôpico Johann Peter Gustav Lejeune Dirichlet
Na verdade ela começou com uma prata.
Ah,o Ciprian Manolescu sobreviveu ao problema
mais dificil de todos os tempos
 --- Paulo Santa Rita [EMAIL PROTECTED]
escreveu:  Ola Pessoal,
 
 No endereco :
 

http://vyasa.math.iisc.ernet.in/PEOPLE/halloffame.html
 
 Voces podem ver varios fatos curiosos
 relacionados a IMO. Por exemplo, la 
 voces poderao ver os medalhistas imo que
 conseguiram tambem ter uma medalha 
 fields ( O Yoccoz, amigo do Prof Gugu e um
 deles : IMO em 1974 e FIELDS em 
 1994 ), estudantes que competiram durante 3
 anos consecutivos e conseguiram 
 3 medalhas de ouro, estudantes com 5 medalhas (
 3 ouros + duas outras entre 
 bronze e prata ) e assim sucessivamente.
 
 O que me pareceu o desempenho mais notavel foi
 o de  uma atleta mulher que 
 saindo de um pais ainda altamente
 preconceituoso ( URSS ) contra as mulheres 
 disputou 3 anos (1989 a 1991 ) consecutivos e
 conseguiu 3 medalhas de ouro : 
   Evgenija Malinnikova. Essa notavel estudante
 venceu muito mais que tres 
 IMO's. Venceu a enorme pobreza de suas origens
 e o forte preconceito contra 
 as mulheres que ainda existe em seu pais.
 
 Muitos outros fatos existem. Vale a pena dar
 uma olhada !
 
 Um Abraco a Todos !
 Paulo Santa Rita
 7,2127,190703
 

_
 MSN Messenger: converse com os seus amigos
 online.  
 http://messenger.msn.com.br
 

=
 Instruções para entrar na lista, sair da lista
 e usar a lista em

http://www.mat.puc-rio.br/~nicolau/olimp/obm-l.html

= 

___
Yahoo! Mail
Mais espaço, mais segurança e gratuito: caixa postal de 6MB, antivírus, proteção 
contra spam.
http://br.mail.yahoo.com/
=
Instruções para entrar na lista, sair da lista e usar a lista em
http://www.mat.puc-rio.br/~nicolau/olimp/obm-l.html
=


[obm-l] IMO 2003 -- Problema 1

2003-07-20 Por tôpico Fábio Dias Moreira
Oi pessoal,

Acabei de chegar do Japão, e dei uma olhada rápida nos emails da lista. Eu li as 
soluções do P1 da IMO, que estão na linha da solução do Alex. Eu acabei descobrindo 
sem querer na prova que o problema é muito folgado, se as escolhas dos ti's forem 
apropriadas.

Tome dA = {x-y|xy, x e y elementos de A}. |dA| = 5050.

Tome t1 = 1. Marque como proibidos todos os inteiros da forma 1+dA, i.e. da forma 1+x 
com x em dA. Tome o menor elemento de S que ainda não foi proibido e chame-o de t2. 
Proíba todo mundo da forma t2+dA. Tome o menor elemento de S não proibido de t3. (...)

É impossível que tj-ti esteja em dA, com ji, pois então tjti, logo tj = ti+x, x em 
dA, *absurdo*, pois então tj teria sido proibido, por construção. Logo basta verificar 
que todos os ti's estão 

Mas há no máximo 5050 proibidos e 1 escolhido antes de t2, logo t2 = 5051; t3 = 
10102; ...; t100 = 5051*99 = 500049  10^6.

(Um subproblema: Seja A um conjunto de inteiros positivos tal que |dA| = n. Quanto 
vale k, o valor mínimo de max(A)? Eu acho que n = 5050 = k  10^6, mas não pensei a 
fundo no problema. Se isso for verdade, o problema é mais folgado ainda.)

[]s,

-- 
Fábio ctg \pi Dias Moreira

=
Instruções para entrar na lista, sair da lista e usar a lista em
http://www.mat.puc-rio.br/~nicolau/olimp/obm-l.html
=


[obm-l] Re: [obm-l] IMO - P1

2003-07-19 Por tôpico yurigomes

Oi Marcio,
 Soh hj eu li seu email, depois que eu tbm consegui fazer a questão. 
 Tem apenas um detalhe que vc não observou: os t_i´s devem ser distintos,
pq senão os dois conjuntos seriam iguais. 
 Seguindo a sua notação, sendo D_i=(D+ t_i)U(t_i- D), temos |D_i|= 2.5050.
O t_(i+1) deve ser escolhido em 
   T = S\(S_1 U...U S_i U {t_1, t_2,...,t_i})
  Olha como o problema é impressionante: para garantir que t_100 pode ser
escolhido, devemos ter T não-vazio. Ora, 
|S_1 U...U S_99 U {t_1, t_2,...,t_99}|= |S_1|+...+ |S_99|+ 99=
  99.2.5050+ 99= 00+ 99= 99  100  ()
   Os números foram muitos bem escolhidos, e o problema ainda não perdeu
a elegância com números feios! NOvamente, parabéns Gugu.
  Ateh mais, 
  Yuri
-- Mensagem original --

Acho que consegui fazer  o 1o. Confiram ai e vejam se tem algum furo.
O
2o eu realmente nao estou conseguindo.. Estou com alguma esperanca de fazer
o 5.. (o 3 eu tentei tmb, mas minhas contas estao muito grandes). Mandem
seus comentarios sobre a prova!
P1:
Note que (Ai inter Aj) != vazio sse existirem m,n tais que a_m + t_i
=
a_n + t_j , i.e, a_m - a_n = t_j - t_i.
Vamos construir os t's indutivamente garantindo que isso nao acontece.
Existem binomial (101,2) = 5050 diferencas possiveis no conjunto A. Chame
de
D={D1,D2,...D5050} o conjunto dessas diferencas (claro que algumas delas
podem ser iguais, mas temos |D| = 5050).
1. Escolha um t1 qualquer de S.
2. Agora quero garantir que t2-t1 e t1-t2 nao estao em D. Para isso, basta
escolher um elemento de S que nao esteja em
X1 = {t1+D1,t1+D2,...,t1+D5050}U{t1-D1, t1-D2,...,t1-D5050}. (pq se t2-t1
esta em D, entao t2=t1+Dk para algum k).
Isso eh facil pq |X1|=2.5050  |S|.

3. Agora vou escolher t3 em S garantindo que t3-t1, t1-t3, t3-t2, t2-t3
nao
estao em D.
Para isso, t3 nao pode estar em X1 e tmb nao pode estar em
X2 = {t2+D1,t2+D2,...,t2+D5050}U{t2-D1, t2-D2,...,t2-D5050}.
Isso eh facil, pq |X1 U X2| = 4.5050  |S|

Em geral, depois de escolhidos t1,t2,...,t_k-1, vou escolher t_k em S de
modo que ele nao esteja em nenhum dos conjuntos X1,X2,...,X_(k-1).
Para k=100, isso eh sempre possivel, pq |X1 U X2 U ... U X_(k-1)| =
2*(k-1)*5050 = 2*99*5050 = 00  10^6 = |S|.
(obs: X_s = {ts + D}U{ts-D}, na notacao usuao de x+A onde x eh um elemento
e
A um conjunto).


Pronto. Foram escolhidos 100 t's tal que nao existe uma quadrupla (m,n,i,j)
tq a_m - a_n = t_j - t_i. (pois t_j - t_i esta sempre fora de D), e portanto
nunca se tem a_m + t_i = a_n + t_j, ou seja, as intersecoes sao todas vazias
de fato.

Abracos.



- Original Message -
From: [EMAIL PROTECTED]
To: [EMAIL PROTECTED]
Cc: [EMAIL PROTECTED]; [EMAIL PROTECTED]
Sent: Monday, July 14, 2003 3:38 PM
Subject: [obm-l] Problemas da IMO




 Prova da IMO retirada do Site http://www.mathlinks.go.ro/

 O Problema 1 é nois que mandou...


 First Day - 44th IMO 2003 Japan

 1. Let A be a 101-element subset of the set S={1,2,3,...,100}. Prove
that
 there exist numbers t_1, t_2, ..., t_{100} in S such that the sets

 Aj = { x + tj | x is in A } for each j = 1, 2, ..., 100

 are pairwise disjoint.


 2. Find all pairs of positive integers (a,b) such that the number

 a^2 / ( 2ab^2-b^3+1) is also a positive integer.

 3. Given is a convex hexagon with the property that the segment connecting
the
 middle points of each pair of opposite sides in the hexagon is  sqrt(3)
/
2
 times the sum of those sides' sum.

 Prove that the hexagon has all its angles equal to 120.


 Second Day - 44th IMO 2003 Japan

 4. Given is a cyclic quadrilateral ABCD and let P, Q, R be feet of the
 altitudes from D to AB, BC and CA respectively. Prove that if PR = RQ
then
the
 interior angle bisectors of the angles  ABC and  ADC are concurrent
on
AC.

 5. Let x1 = x2 = ... = xn be real numbers, n2.

 a) Prove the following inequality:

 (sum  ni,j=1 | xi - xj | ) 2 = 2/3 ( n^2 - 1 )sum ni,j=1 ( xi - xj)^2

 b) Prove that the equality in the inequality above is obtained if and
only
if
 the sequence (xk) is an arithemetical progression.

 6. Prove that for each given prime p there exists a prime q such that
n^p - p
 is not divisible by q for each positive integer n.



 -
 This mail sent through IMP: http://horde.org/imp/
 =
 Instruções para entrar na lista, sair da lista e usar a lista em
 http://www.mat.puc-rio.br/~nicolau/olimp/obm-l.html
 =


=
Instruções para entrar na lista, sair da lista e usar a lista em
http://www.mat.puc-rio.br/~nicolau/olimp/obm-l.html
=


[]'s, Yuri
ICQ: 64992515


--
Use o melhor sistema de busca da Internet
Radar UOL - http://www.radaruol.com.br




Re: [obm-l] Re: [obm-l] IMO - P1

2003-07-19 Por tôpico Marcio Afonso A. Cohen
É verdade! Valeu!
Marcio

- Original Message -
From: [EMAIL PROTECTED]
To: [EMAIL PROTECTED]
Sent: Saturday, July 19, 2003 4:49 PM
Subject: [obm-l] Re: [obm-l] IMO - P1



 Oi Marcio,
  Soh hj eu li seu email, depois que eu tbm consegui fazer a questão.
  Tem apenas um detalhe que vc não observou: os t_i´s devem ser distintos,
 pq senão os dois conjuntos seriam iguais.
  Seguindo a sua notação, sendo D_i=(D+ t_i)U(t_i- D), temos |D_i|=
2.5050.
 O t_(i+1) deve ser escolhido em
T = S\(S_1 U...U S_i U {t_1, t_2,...,t_i})
   Olha como o problema é impressionante: para garantir que t_100 pode ser
 escolhido, devemos ter T não-vazio. Ora,
 |S_1 U...U S_99 U {t_1, t_2,...,t_99}|= |S_1|+...+ |S_99|+ 99=
   99.2.5050+ 99= 00+ 99= 99  100  ()
Os números foram muitos bem escolhidos, e o problema ainda não perdeu
 a elegância com números feios! NOvamente, parabéns Gugu.
   Ateh mais,

=
Instruções para entrar na lista, sair da lista e usar a lista em
http://www.mat.puc-rio.br/~nicolau/olimp/obm-l.html
=


[obm-l] IMO - Curiosidades.

2003-07-19 Por tôpico Paulo Santa Rita
Ola Pessoal,

No endereco :

http://vyasa.math.iisc.ernet.in/PEOPLE/halloffame.html

Voces podem ver varios fatos curiosos relacionados a IMO. Por exemplo, la 
voces poderao ver os medalhistas imo que conseguiram tambem ter uma medalha 
fields ( O Yoccoz, amigo do Prof Gugu e um deles : IMO em 1974 e FIELDS em 
1994 ), estudantes que competiram durante 3 anos consecutivos e conseguiram 
3 medalhas de ouro, estudantes com 5 medalhas ( 3 ouros + duas outras entre 
bronze e prata ) e assim sucessivamente.

O que me pareceu o desempenho mais notavel foi o de  uma atleta mulher que 
saindo de um pais ainda altamente preconceituoso ( URSS ) contra as mulheres 
disputou 3 anos (1989 a 1991 ) consecutivos e conseguiu 3 medalhas de ouro : 
 Evgenija Malinnikova. Essa notavel estudante venceu muito mais que tres 
IMO's. Venceu a enorme pobreza de suas origens e o forte preconceito contra 
as mulheres que ainda existe em seu pais.

Muitos outros fatos existem. Vale a pena dar uma olhada !

Um Abraco a Todos !
Paulo Santa Rita
7,2127,190703
_
MSN Messenger: converse com os seus amigos online.  
http://messenger.msn.com.br

=
Instruções para entrar na lista, sair da lista e usar a lista em
http://www.mat.puc-rio.br/~nicolau/olimp/obm-l.html
=


Re: [obm-l] IMO - Problema 2

2003-07-18 Por tôpico Paulo Santa Rita
Oi Prof Gugu !
Tudo Legal ?
Conforme voce diz, usualmente. Mas tenho certeza que voce sabe que muitos 
bons livros tratam dos casos para j qualquer. Ai no IMPA, com certeza tem, 
porque eu ja vi. Vou divulgar aqui na lista ao menos uma livro elementar 
sobre equacoes diofantinas que trata deste casos.

Vou escrever a solucao e te envio.

Carissimo Prof, francamente acho que as solucoes da IMO nao deveriam ser 
enviadas para a lista pelos Mestres que orientam nossos atletas, pois e 
natural que os estudantes se sintam desestimulados em buscar uma solucao 
propria quando sabemos que a solucao ja esta divulgada. Por outro lado, para 
o Sr ou para os Profs Nicolau, Morgado, Wagner e os outros orientadores, 
resolver as questoes da IMO nao significa ou indica nada, enquanto que para 
um estudante, olimpico ou nao, uma solucao propria representa muito.

Essa e a minha impressao ! Mas, reconheco que o Prof e demais orientadores 
sao muito mais experientes, teem muito mais conhecimentos, razao pela qual 
modifico imediatamente esta minha ideia se o Prof achar que isso e bobeira. 
Eu falei com o Prof Nicolau neste sentido.

Mudando de assunto. O Prof pensou na quetao 3 ?

Ontem, almocando no restaurante, eu pensei nela ( e a comida ficou fria e eu 
nao almocei ! ), mas nao consegui resolver naquele tempo. Observei o 
seguinte :

IMAGINADO o hexagono ABCDEF no primeiro quadrante, IMAGINANDO os vetores A, 
B, C, D, E, F, e IMAGIANANDO o segmento MN que une os pontos medios de AB e 
DE ( M em AB e N em DE ), segue que :

M = 1/2 * (A+B) e N = 1/2 * (D+E)
MN = 1/2 * modulo( (A+B) - (D+E) )
A propriedade diz que :

MN = sqrt(3)/2 * (AB+DE)

ou seja :

1/2 * modulo( (A+B) - (D+E) ) = sqrt(3)/2  * ( modulo(B-A) + modulo(D-E) )
modulo( (A+B) - (D+E) ) = sqrt(3)*( modulo(B-A) + modulo(D-E) )
Isto pode ser colocado assim :
modulo( (A - E) + (B - D) ) = sqrt(3)/2  * ( modulo( B - A) + modulo( D - E) 
)
Evidentemente que para os demais pares de lados opostos valera uma relacao 
semelhante.

Os vetores A-E e B-D e os outros derivados das outras duas relacoes dos 
outros dois pares de lados opostos sao subtendidos pelos angulos internos do 
hexagono, segue que seus modulos podem ser expressos em funcao de alguma 
funcao trigonometrica aplicada a estes angulos internos.

E verdade que se todos os angulos internos valerem 120 graus entao os pares 
de lados opostos sao paralelos e a relacao vale. Suponha agora que um dos 
angulos nao e 120, entao, aplicando o principio da casa dos pombos aos 
demais, segue que havera algum outro tambem diferente de 120.

Eu parei aqui. Havendo tempo ( Ah se eu tivesse tempo so pra estudar ! ) vou 
prosseguir por este caminho.

Um Abracao pro Prof !
Paulo Santa Rita
6,1029,180703
From: [EMAIL PROTECTED]
Reply-To: [EMAIL PROTECTED]
To: [EMAIL PROTECTED]
Subject: Re: [obm-l] IMO - Problema 2
Date: Thu, 17 Jul 2003 18:41:48 -0300
Caro Paulo,
Usualmente o termo equacao de Pell se refere ao caso j=1 (e o 
coeficiente
de b^2 nao depende nem de a nem de b). Nao entendi como concluir uma 
solucao na
linha que voce propos. Por outro lado eu consegui (depois de tropecar um 
pouco)
achar uma solucao, que reproduzo abaixo, depois de algum espaco, para nao
atrapalhar quem queira pensar mais no problema.

...



...



...



...



...



...



Vamos la':

   Se b=1 o problema e' achar todos os a tais que a^2/2a=a/2 e' inteiro. 
Isso
nos da' as solucoes {(a,1),a par}. Vamos supor agora b=2.
   Se 2ab^2-b^3+1 divide a^2 entao tambem divide 
a^2.(2b^2)-a(2ab^2-b^3+1)=
=a(b^3-1) e (1-b^3)(2ab^2-b^3+1)+(2b^2)(a(b^3-1))=(1-b^3)^2. Sejam entao
d=mdc(a,1-b^3), a=kd, 1-b^3=ud. Temos que mdc(k,u)=1 e que 2ab^2-b^3+1 
divide
mdc(a^2,(1-b^3)^2)=d^2, ou seja, d(2kb^2+u) divide d^2, e logo 2kb^2+u 
divide
d. Portanto, tambem temos que 2kb^2+u divide b(2kb^2+u)+2kud=ub+2k. Temos 
agora
dois casos:
   i)d=kb^2. Entao |u|=|(1-b^3)/d|  b^3/(kb^2)=b/k. Nesse caso,
|2kb^2+u|=2kb^2-|u|  2kb^2-b/k, enquanto |ub+2k|  b^2/k+2k. Como b=2, 
2kb^2-
b/k=kb^2+b(kb-1/k)=b^2/k+2(2k-1/k)=b^2/k+2k, donde |2kb^2+u|  |ub+2k|, e
portanto devemos ter ub+2k=0, donde b(1-b^3)+2a=dub+2dk=0, e logo 
a=b(b^3-1)/2.
Isso nos da' a^2/(2ab^2-b^3+1)=b^2/4, que e' inteiro quando b e' par. Isso 
nos
da' (todas) as solucoes nesse caso i): {(b(b^3-1)/2,b), b par}.
   ii)d  kb^2. Aqui, como 2kb^2+u divide d, devemos ter 
kb^2|d|=|2kb^2+u|=
=2kb^2-|u|, donde |u|kb^2. Assim, temos ab^2=kb^2d  |ud|=b^3-1  b^3, 
donde
a  b, ou seja, b=a+1. Como 2ab^2-b^3+1 e' congruente a 1 modulo b^2, ou
2ab^2-b^3+1=1 ou |2ab^2-b^3+1|=|1-b^2|=b^2-1=(a+1)^2-1=a^2+2a  a^2, mas,
nesse caso, 2ab^2-b^3+1 nao pode dividir a^2. Assim, devemos ter 
2ab^2-b^3+1=1,
donde 2ab^2=b^3, e b=2a. Isso nos da' as solucoes do caso ii): {(a,2a)}.

   Conclusao: as solucoes do problema sao dadas por: {(a,1), a inteiro 
positivo
par}, {(b(b^3-1)/2,b), b inteiro positivo par} ou {(a,2a), a inteiro 
positivo}.
   Abracos,
Gugu

Quoting Paulo Santa Rita [EMAIL PROTECTED]:

 Ola Cicero

Re: [obm-l] IMO - Problema 2

2003-07-17 Por tôpico gugu
 ingerir periodicamente.
 
 Eu percebi que a prova da infinidade de solucoes, dada uma solucao 
 particular, e forcada. E muito mais uma justificativa que uma solucao. Deve
 
 haver uma forma de deduzir a sua infinidade de solucoes a partir de uma 
 conceituacao mais geral, mais eu ainda nao consegui encontrar isso na 
 literatura matematica, por mais que tenha forcejado neste sentido.
 
 Foi sem duvida uma descoberta notavel, mas eu sinto que neste mar existe 
 muito mais coisas a serem descobertas. Uma prova indireta disso e a equacao
 
 de Euler : a^3 = b^2 + 2. Ela tem uma unica solucao inteira e resultou de um
 
 trabalho do Euler sobre um problema proposto pelo Fermat.
 
 Bom, eu vou ficando por aqui senao vou escrever muito e o trabalho me chama
 
 e os sistemas precisao ser concluidos. Mas o que eu queria dizer e que esse
 
 tema e facisnante e que todo Matematico Serio deveria trata-lo com carinho e
 
 devocao.
 
 FECHA PARENTESES
 
 Um Abraco a Todos
 Paulo Santa Rita
 4,2031,160703
 
 
 
 
 From: [EMAIL PROTECTED]
 Reply-To: [EMAIL PROTECTED]
 To: [EMAIL PROTECTED]
 Subject: [obm-l] IMO
 Date: Wed, 16 Jul 2003 18:00:24 -0300
 
 
 
 Olá galera,
 
 O Problema 4 realmente é muito simples, alguns conhecimentos de reta de
 simpson e lei dos senos resolvem o problema. Mas estou agora pensando no
 2, tive a´idéia seguinte:
 
 a^2 = 2ab^2 - b^3 + 1, e dai ver que é uma parábola em a e o delta tem
 que ser  0. Será uma boa idéia?? Alguém fez o problema 2??
 Cícero
 
 
 
 --
 Use o melhor sistema de busca da Internet
 Radar UOL - http://www.radaruol.com.br
 
 


-
This mail sent through IMP: http://horde.org/imp/
=
Instruções para entrar na lista, sair da lista e usar a lista em
http://www.mat.puc-rio.br/~nicolau/olimp/obm-l.html
=


[obm-l] IMO

2003-07-16 Por tôpico ciceroth


Olá galera,

O Problema 4 realmente é muito simples, alguns conhecimentos de reta de
simpson e lei dos senos resolvem o problema. Mas estou agora pensando no
2, tive a´idéia seguinte:

a^2 = 2ab^2 - b^3 + 1, e dai ver que é uma parábola em a e o delta tem
que ser  0. Será uma boa idéia?? Alguém fez o problema 2??
Cícero



--
Use o melhor sistema de busca da Internet
Radar UOL - http://www.radaruol.com.br



=
Instruções para entrar na lista, sair da lista e usar a lista em
http://www.mat.puc-rio.br/~nicolau/olimp/obm-l.html
=


Re: [obm-l] IMO - P1

2003-07-15 Por tôpico Marcio Afonso A. Cohen
Acho que consegui fazer  o 1o. Confiram ai e vejam se tem algum furo. O
2o eu realmente nao estou conseguindo.. Estou com alguma esperanca de fazer
o 5.. (o 3 eu tentei tmb, mas minhas contas estao muito grandes). Mandem
seus comentarios sobre a prova!
P1:
Note que (Ai inter Aj) != vazio sse existirem m,n tais que a_m + t_i =
a_n + t_j , i.e, a_m - a_n = t_j - t_i.
Vamos construir os t's indutivamente garantindo que isso nao acontece.
Existem binomial (101,2) = 5050 diferencas possiveis no conjunto A. Chame de
D={D1,D2,...D5050} o conjunto dessas diferencas (claro que algumas delas
podem ser iguais, mas temos |D| = 5050).
1. Escolha um t1 qualquer de S.
2. Agora quero garantir que t2-t1 e t1-t2 nao estao em D. Para isso, basta
escolher um elemento de S que nao esteja em
X1 = {t1+D1,t1+D2,...,t1+D5050}U{t1-D1, t1-D2,...,t1-D5050}. (pq se t2-t1
esta em D, entao t2=t1+Dk para algum k).
Isso eh facil pq |X1|=2.5050  |S|.

3. Agora vou escolher t3 em S garantindo que t3-t1, t1-t3, t3-t2, t2-t3 nao
estao em D.
Para isso, t3 nao pode estar em X1 e tmb nao pode estar em
X2 = {t2+D1,t2+D2,...,t2+D5050}U{t2-D1, t2-D2,...,t2-D5050}.
Isso eh facil, pq |X1 U X2| = 4.5050  |S|

Em geral, depois de escolhidos t1,t2,...,t_k-1, vou escolher t_k em S de
modo que ele nao esteja em nenhum dos conjuntos X1,X2,...,X_(k-1).
Para k=100, isso eh sempre possivel, pq |X1 U X2 U ... U X_(k-1)| =
2*(k-1)*5050 = 2*99*5050 = 00  10^6 = |S|.
(obs: X_s = {ts + D}U{ts-D}, na notacao usuao de x+A onde x eh um elemento e
A um conjunto).


Pronto. Foram escolhidos 100 t's tal que nao existe uma quadrupla (m,n,i,j)
tq a_m - a_n = t_j - t_i. (pois t_j - t_i esta sempre fora de D), e portanto
nunca se tem a_m + t_i = a_n + t_j, ou seja, as intersecoes sao todas vazias
de fato.

Abracos.



- Original Message -
From: [EMAIL PROTECTED]
To: [EMAIL PROTECTED]
Cc: [EMAIL PROTECTED]; [EMAIL PROTECTED]
Sent: Monday, July 14, 2003 3:38 PM
Subject: [obm-l] Problemas da IMO




 Prova da IMO retirada do Site http://www.mathlinks.go.ro/

 O Problema 1 é nois que mandou...


 First Day - 44th IMO 2003 Japan

 1. Let A be a 101-element subset of the set S={1,2,3,...,100}. Prove
that
 there exist numbers t_1, t_2, ..., t_{100} in S such that the sets

 Aj = { x + tj | x is in A } for each j = 1, 2, ..., 100

 are pairwise disjoint.


 2. Find all pairs of positive integers (a,b) such that the number

 a^2 / ( 2ab^2-b^3+1) is also a positive integer.

 3. Given is a convex hexagon with the property that the segment connecting
the
 middle points of each pair of opposite sides in the hexagon is  sqrt(3) /
2
 times the sum of those sides' sum.

 Prove that the hexagon has all its angles equal to 120.


 Second Day - 44th IMO 2003 Japan

 4. Given is a cyclic quadrilateral ABCD and let P, Q, R be feet of the
 altitudes from D to AB, BC and CA respectively. Prove that if PR = RQ then
the
 interior angle bisectors of the angles  ABC and  ADC are concurrent on
AC.

 5. Let x1 = x2 = ... = xn be real numbers, n2.

 a) Prove the following inequality:

 (sum  ni,j=1 | xi - xj | ) 2 = 2/3 ( n^2 - 1 )sum ni,j=1 ( xi - xj)^2

 b) Prove that the equality in the inequality above is obtained if and only
if
 the sequence (xk) is an arithemetical progression.

 6. Prove that for each given prime p there exists a prime q such that
n^p - p
 is not divisible by q for each positive integer n.



 -
 This mail sent through IMP: http://horde.org/imp/
 =
 Instruções para entrar na lista, sair da lista e usar a lista em
 http://www.mat.puc-rio.br/~nicolau/olimp/obm-l.html
 =


=
Instruções para entrar na lista, sair da lista e usar a lista em
http://www.mat.puc-rio.br/~nicolau/olimp/obm-l.html
=


[obm-l] Re: [obm-l] IMO, QUEBRA-CABEÇAS, ALGORITMOS, ETC

2003-02-17 Por tôpico Cláudio \(Prática\)



Oi Fael:

Primeiro, uma pequena correção: o cubo é de Rubik 
e, se você estiver interessado, existem vários sites descrevendo estratégias e 
até com análises matemáticas do cubo. A área da matemática usada para estas 
análises é a teoria dos grupos, onde há uma grande atividade de pesquisa em 
andamento atualmente e onde ainda restam muitos problemas em aberto. Assim, você 
mesmo deu um exemplo da correlação entre matemática recreativa (cubo de Rubik) e 
matemática avançada (teoria dos grupos) usada para resolver um problema 
recreativo.

Sobre a sua pergunta, acho que qualquer um que 
tenha facilidade para matemática haverá de ter facilidade na solução de 
problemas recreativos. No entanto, como distinguir um problema "recreativo" de 
um que é tema de pesquisa "séria"? Aqui mesmo na lista, já apareceram 
vários que podem ser classificados de um jeito ou de outro. Por exemplo, esse do 
Paulo Santa Rita:

Em uma reuniao existem exatamente 201 pessoas de 5 nacionalidades 
diferentes. Sabe-se que em cada grupo de 6 pessoas, ao menos duas tem a 
mesma idade. Demonstrar que existem ao menos 5 pessoas do mesmo pais, da 
mesma idade e do mesmo sexo.

envolve um conceito chamado princípio das gavetas, 
que diz o seguinte: se existem N+1 meias que devem ser colocadas em N gavetas, 
então alguma gaveta irá conter pelo menos duas meias. Existe uma teoria 
matemática (teoria de Ramsey) que estuda extensões e variações deste princípio, 
ainda tem vários problemas sem solução, e é objeto de pesquisa em várias 
universidades.

(Paulo - espero que você não se ofenda por eu ter 
classificado o seu problema como recreativo, mas como eu não sou aluno (no 
sentido estrito, isto é, matriculado em alguma instituição), professor ou 
pesquisador, pra mim matemática é um hobby e, portanto,todos os problemas 
são recreativos).

Um abraço,
Claudio.



  - Original Message - 
  From: 
  [EMAIL PROTECTED] 
  
  To: [EMAIL PROTECTED] 
  Sent: Saturday, February 15, 2003 6:09 
  PM
  Subject: [obm-l] IMO, QUEBRA-CABEÇAS, 
  ALGORITMOS, ETC
  Olá pessoal, Sabemos que existem 
  algoritmos não só para a resolução mas tbém para dizer qual a quantidade 
  mínima giros que poderiam ser dados em um cubo Rubrick (ou cubo mágico) sendo 
  dada uma disposição inicial do mesmo. Sabemos que existem vários 
  quebra-cabeças como o cubo Rubrick e sabemos tbém que vários deles podem ser 
  criados. Como gosto muito de matemática recreacional gostaria de saber a 
  correlação entre matemática avançada ou geral e matemática recreacional. 
  Quando digo correlação falo no sentido estritamente estatístico, ou seja, todo 
  indivíduo especializado em matemática geral, como muitos aqui, é tbém 
  altamente capaz em questões de matemática recreacional ? Ou uma coisa não tem 
  a ver com a outra? Voltando aos quebra-cabeças estilo cubo rubrick. Se fosse 
  criada uma etapa na IMO, por exemplo, para a criação de algoritmos e resolução 
  destes tipos de quebra-cabeças seria um "filtro" (seleção) muito rigoroso? Ou 
  seria fácil para os mais tarimbados ? ICQ: 337140512 



[obm-l] IMO, QUEBRA-CABEÇAS, ALGORITMOS, ETC

2003-02-15 Por tôpico Faelccmm
Olá pessoal,

Muitos já conhecem o site mas os que não conhecem e se interessam pelo assunto aqui vai a dica:

http://mathworld.wolfram.com/topics/RecreationalMathematics.html


[obm-l] IMO, QUEBRA-CABEÇAS, ALGORITMOS, ETC

2003-02-15 Por tôpico Faelccmm
Olá pessoal,

Sabemos que existem algoritmos não só para a resolução mas tbém para dizer qual a quantidade mínima giros que poderiam ser dados em um cubo Rubrick (ou cubo mágico) sendo dada uma disposição inicial do mesmo. Sabemos que existem vários quebra-cabeças como o cubo Rubrick e sabemos tbém que vários deles podem ser criados. Como gosto muito de matemática recreacional gostaria de saber a correlação entre matemática avançada ou geral e matemática recreacional. Quando digo correlação falo no sentido estritamente estatístico, ou seja, todo indivíduo especializado em matemática geral, como muitos aqui, é tbém altamente capaz em questões de matemática recreacional ? Ou uma coisa não tem a ver com a outra? Voltando aos quebra-cabeças estilo cubo rubrick. Se fosse criada uma etapa na IMO, por exemplo, para a criação de algoritmos e resolução destes tipos de quebra-cabeças seria um "filtro" (seleção) muito rigoroso? Ou seria fácil para os mais tarimbados ?

ICQ: 337140512


Re: [obm-l] IMO(ih!!!!)

2003-02-10 Por tôpico Johann Peter Gustav Lejeune Dirichlet
Antes que isso se alastre mais do que ja esta,deixe-me explicar:essas mensagens nao eram para a lista.Eu deveria manda-las para mim mesmo (o Word foi apagado de meu computador pelo tosco do meu irmao) e depois imprimi-las (pois minha impressora esta um lixo),e acabei mandando para a Lista por engano.Desculpe-me pelo Junk Mail,ta?
Ass.:JohannTRANSIRE SVVM PECTVS MVNDOQUE POTIRE
CONGREGATI EX TOTO ORBE MATHEMATICI OB SCRIPTA INSIGNIA TRIBVERE
Fields Medal(John Charles Fields)Busca Yahoo! 
O serviço de busca mais completo da Internet. O que você pensar o Yahoo! encontra.

Re: [obm-l] IMO

2003-02-09 Por tôpico amurpe
 Acho que nao tem muito a ver voce ficar inundando a lis
ta com problemas resolvidos.. A maioria das pessoas aqui 
conhece o site do Kalva, e lá há diversos problemas resol
vidos, de diversos níveis de dificuldade.. 
   - Original Message - 
   From: Johann Peter Gustav Lejeune Dirichlet 
   To: [EMAIL PROTECTED] 
   Sent: Thursday, February 06, 2003 3:26 PM
   Subject: [obm-l] IMO
 
 
   Problem 3
 
   The set of all positive integers is the union of two 
disjoint subsets {f(1), f(2), f(3), ... }, {g(1), g(2), g
(3), ... }, where f(1)  f(2)  f(3)  ..., and g(1)  g
(2)  g(3)  ... , and g(n) = f(f
(n)) + 1 for n = 1, 2, 3, ... . Determine f(240). 
 
 
   Solution
 Alô pessoal , gosraia de saber o site do KAlva .por 
favor me enviem .um abraço.Amurpe
 
   Let F = {f(1), f(2), f(3), ... }, G = {g(1), g(2), g
(3), ... }, Nn = {1, 2, 3, ... , n}. f(1) = 1, so f(f
(1)) = 1 and hence g
(1) = 2. So 1 is not in G, and hence must be in F. It mu
st be the smallest element of F and so f(1) = 1. Hence g
(1) = 2. We can never have two successive integers n and 
n+1 in G, because if g(m) = n+1, then f
(something) = n and so n is in F and G. Contradiction. In
 particular, 3 must be in F, and so f(2) = 3. 
 
   Suppose f(n) = k. Then g(n) = f(k) + 1. So |Nf(k)
+1 Ç G| = n. But |Nf(k)+1 Ç F| = k, so n + k = f
(k) + 1, or f(k) = n + k - 1. Hence g
(n) = n + k. So n + k + 1 must be in F and hence f
(k+1) = n + k + 1. This so given the value of f for n we 
can find it for k and k+1. 
 
   Using k+1 each time, we get, successively, f
(2) = 3, f(4) = 6, f(7) = 11, f(12) = 19, f(20) = 32, f
(33) = 53, f(54) = 87, f(88) = 142, f(143) = 231, f
(232) = 375, which is not much help. Trying again with k,
 we get: f(3) = 4, f(4) = 6, f(6) = 9, f(9) = 14, f
(14) = 22, f(22) = 35, f(35) = 56, f(56) = 90, f
(90) = 145, f
(145) = 234. Still not right, but we can try backing up s
lightly and using k+1: f
(146) = 236. Still not right, we need to back up further:
 f(91) = 147, f(148) = 239, f(240) = 388. 
 
 
 
 
 
 
 
 ---
---
   Busca Yahoo! 
   O serviço de busca mais completo da Internet. O que v
ocê pensar o Yahoo! encontra.
 

 
__
E-mail Premium BOL
Antivírus, anti-spam e até 100 MB de espaço. Assine já!
http://email.bol.com.br/


=
Instruções para entrar na lista, sair da lista e usar a lista em
http://www.mat.puc-rio.br/~nicolau/olimp/obm-l.html
O administrador desta lista é [EMAIL PROTECTED]
=



Re: [obm-l] IMO

2003-02-09 Por tôpico Henrique Lima Santana
a hp eh a seguinte,
www.kalva.demon.co.uk
falou
henrique






From: amurpe [EMAIL PROTECTED]
Reply-To: [EMAIL PROTECTED]
To: [EMAIL PROTECTED]
Subject: Re: [obm-l] IMO
Date: Sun,  9 Feb 2003 08:37:58 -0200

 Acho que nao tem muito a ver voce ficar inundando a lis
ta com problemas resolvidos.. A maioria das pessoas aqui
conhece o site do Kalva, e lá há diversos problemas resol
vidos, de diversos níveis de dificuldade..
   - Original Message -
   From: Johann Peter Gustav Lejeune Dirichlet
   To: [EMAIL PROTECTED]
   Sent: Thursday, February 06, 2003 3:26 PM
   Subject: [obm-l] IMO


   Problem 3

   The set of all positive integers is the union of two
disjoint subsets {f(1), f(2), f(3), ... }, {g(1), g(2), g
(3), ... }, where f(1)  f(2)  f(3)  ..., and g(1)  g
(2)  g(3)  ... , and g(n) = f(f
(n)) + 1 for n = 1, 2, 3, ... . Determine f(240).


   Solution
 Alô pessoal , gosraia de saber o site do KAlva .por
favor me enviem .um abraço.Amurpe

   Let F = {f(1), f(2), f(3), ... }, G = {g(1), g(2), g
(3), ... }, Nn = {1, 2, 3, ... , n}. f(1) = 1, so f(f
(1)) = 1 and hence g
(1) = 2. So 1 is not in G, and hence must be in F. It mu
st be the smallest element of F and so f(1) = 1. Hence g
(1) = 2. We can never have two successive integers n and
n+1 in G, because if g(m) = n+1, then f
(something) = n and so n is in F and G. Contradiction. In
 particular, 3 must be in F, and so f(2) = 3.

   Suppose f(n) = k. Then g(n) = f(k) + 1. So |Nf(k)
+1 Ç G| = n. But |Nf(k)+1 Ç F| = k, so n + k = f
(k) + 1, or f(k) = n + k - 1. Hence g
(n) = n + k. So n + k + 1 must be in F and hence f
(k+1) = n + k + 1. This so given the value of f for n we
can find it for k and k+1.

   Using k+1 each time, we get, successively, f
(2) = 3, f(4) = 6, f(7) = 11, f(12) = 19, f(20) = 32, f
(33) = 53, f(54) = 87, f(88) = 142, f(143) = 231, f
(232) = 375, which is not much help. Trying again with k,
 we get: f(3) = 4, f(4) = 6, f(6) = 9, f(9) = 14, f
(14) = 22, f(22) = 35, f(35) = 56, f(56) = 90, f
(90) = 145, f
(145) = 234. Still not right, but we can try backing up s
lightly and using k+1: f
(146) = 236. Still not right, we need to back up further:
 f(91) = 147, f(148) = 239, f(240) = 388.







 ---
---
   Busca Yahoo!
   O serviço de busca mais completo da Internet. O que v
ocê pensar o Yahoo! encontra.



__
E-mail Premium BOL
Antivírus, anti-spam e até 100 MB de espaço. Assine já!
http://email.bol.com.br/


=
Instruções para entrar na lista, sair da lista e usar a lista em
http://www.mat.puc-rio.br/~nicolau/olimp/obm-l.html
O administrador desta lista é [EMAIL PROTECTED]
=



_
The new MSN 8: smart spam protection and 2 months FREE*  
http://join.msn.com/?page=features/junkmail

=
Instruções para entrar na lista, sair da lista e usar a lista em
http://www.mat.puc-rio.br/~nicolau/olimp/obm-l.html
O administrador desta lista é [EMAIL PROTECTED]
=


[obm-l] IMO de Cuba

2003-02-06 Por tôpico Johann Peter Gustav Lejeune Dirichlet
Problem 3
Let x1, x2, ... , xn be real numbers satisfying x12 + x22 + ... + xn2 = 1. Prove that for every integer k = 2 there are integers a1, a2, ... , an, not all zero, such that |ai| = k - 1 for all i, and |a1x1 + a2x2 + ... + anxn| = (k - 1)Ön/(kn - 1). 
 
Solution
This is an application of the pigeon-hole principle. 
Assume first that all xi are non-negative. Observe that the sum of the xi is at most Ön. [This is a well-known variant, (Sum1=i=n xi)2 = n Sum1=i=n xi2, of the AM-GM result. See, for example, Arthur Engel, Problem Solving Strategies, Springer 1998, p163, ISBN 0387982191]. 
Consider the kn possible values of Sum1=i=n bixi, where each bi is an integer in the range [0,k-1]. Each value must lie in the interval [0, k-1 Ön]. Divide this into kn-1 equal subintervals. Two values must lie in the same subinterval. Take their difference. Its coefficients are the required ai. Finally, if any xi are negative, solve for the absolute values and then flip signs in the ai. Busca Yahoo! 
O serviço de busca mais completo da Internet. O que você pensar o Yahoo! encontra.

[obm-l] IMO

2003-02-06 Por tôpico Johann Peter Gustav Lejeune Dirichlet
Problem 6
Prove that there exists a convex 1990-gon such that all its angles are equal and the lengths of the sides are the numbers 12, 22, ... , 19902 in some order. 
 
Solution
By Robin Chapman, Dept of Maths, Macquarie University, Australia 
In the complex plane we can represent the sides as pn2wn, where pn is a permutation of (1, 2, ... , 1990) and w is a primitive 1990th root of unity. 
The critical point is that 1990 is a product of more than 2 distinct primes: 1990 = 2.5.199. So we can write w = -1.a.b, where -1 is primitive 2nd root of unity, a is a primitive 5th root of unity, and b is a primitive 199th root of unity. 
Now given one of the 1990th roots we may write it as (-1)iajbk, where 0  i  2, 0  j  5, 0  k  199 and hence associate it with the integer r(i,j,k) = 1 + 995i + 199j + k. This is a bijection onto (1, 2, ... , 1990). We have to show that the sum of r(i,j,k)2 (-1)iajbk is zero. 
We sum first over i. This gives -9952 x sum of ajbk which is zero, and - 1990 x sum s(j,k) ajbk, where s(j,k) = 1 + 199j + k. So it is sufficient to show that the sum of s(j,k) ajbk is zero. We now sum over j. The 1 + k part of s(j,k) immediately gives zero. The 199j part gives a constant times bk, which gives zero when summed over k. 
Busca Yahoo! 
O serviço de busca mais completo da Internet. O que você pensar o Yahoo! encontra.

[obm-l] IMO

2003-02-06 Por tôpico Johann Peter Gustav Lejeune Dirichlet
Problem 6
Given any real number a  1 construct a bounded infinite sequence x0, x1, x2, ... such that |xn - xm| |n - m|a = 1 for every pair of distinct n, m. 
[An infinite sequence x0, x1, x2, ... of real numbers is bounded if there is a constant C such that |xn|  C for all n.] 
 
Solution
By Marcin Mazur, University of Illinois at Urbana-Champaign 
Let t = 1/2a. Define c = 1 - t/(1 - t). Since a  1, c  0. Now given any integer n  0, take the binary expansion n = Si bi 2i, and define xn = 1/c Sbi0 ti. For example, taking n = 21 = 24 + 22 + 20, we have x21 = (t4 + t2 + t0)/c. We show that for any unequal n, m, |xn - xm| |n - m|a = 1. This solves the problem, since the xn are all positive and bounded by (S tn )/c = 1/(1 - 2t). 
Take k to be the highest power of 2 dividing both n and m. Then |n - m| = 2k. Also, in the binary expansions for n and m, the coefficients of 20, 21, ... , 2k-1 agree, but the coefficients for 2k are different. Hence c |xn - xm| = tk + Sik yi, where yi = 0, ti or - ti. Certainly Sik yi  - Sik ti = tk+1/(1 - t), so c |xn - xm|  tk(1 - t/(1 - t)) = c tk. Hence |xn - xm| |n - m|a  tk 2ak = 1. 
Busca Yahoo! 
O serviço de busca mais completo da Internet. O que você pensar o Yahoo! encontra.

[obm-l] IMO

2003-02-06 Por tôpico Johann Peter Gustav Lejeune Dirichlet
Problem 6
Let p be an odd prime number. How many p-element subsets A of {1, 2, ... , 2p} are there, the sum of whose elements is divisible by p? 
 
Solution
Answer: 2 + (2pCp - 2)/p, where 2pCp is the binomial coefficient (2p)!/(p!p!). 
Let A be a subset other than {1, 2, ... , p} and {p+1, p+2, ... , 2p}. Consider the elements of A in {1, 2, ... , p}. The number r satisfies 0  r  p. We can change these elements to another set of r elements of {1, 2, ... , p} by adding 1 to each element (and reducing mod p if necessary). We can repeat this process and get p sets in all. For example, if p = 7 and the original subset of {1, 2, ... , 7} was {3 , 5}, we get: 
 {3 , 5}, {4, 6}, {5, 7}, {6, 1}, {7, 2}, {1, 3}, {2, 4}. 
The sum of the elements in the set is increased by r each time. So, since p is prime, the sums must form a complete set of residues mod p. In particular, they must all be distinct and hence all the subsets must be different. 
Now consider the sets A which have a given intersection with {p+1, ... , n}. Suppose the elements in this intersection sum to k mod p. The sets can be partitioned into groups of p by the process described above, so that exactly one member of each group will have the sum -k mod p for its elements in {1, 2, ... , p}. In other words, exactly one member of each group will have the sum of all its elements divisible by p. 
There are 2pCp subsets of {1, 2, ... , 2p} of size p. Excluding {1, 2, ... , p} and {p+1, ... , 2p} leaves (2pCp - 2). We have just shown that (2pCp - 2)/p of these have sum divisible by p. The two excluded subsets also have sum divisible by p, so there are 2 + (2pCp - 2)/p subsets in all having sum divisible by p. 
Busca Yahoo! 
O serviço de busca mais completo da Internet. O que você pensar o Yahoo! encontra.

[obm-l] IMO

2003-02-06 Por tôpico Johann Peter Gustav Lejeune Dirichlet
Problem 3
The set of all positive integers is the union of two disjoint subsets {f(1), f(2), f(3), ... }, {g(1), g(2), g(3), ... }, where f(1)  f(2)  f(3)  ..., and g(1)  g(2)  g(3)  ... , and g(n) = f(f(n)) + 1 for n = 1, 2, 3, ... . Determine f(240). 
 
Solution
Let F = {f(1), f(2), f(3), ... }, G = {g(1), g(2), g(3), ... }, Nn = {1, 2, 3, ... , n}. f(1) = 1, so f(f(1)) = 1 and hence g(1) = 2. So 1 is not in G, and hence must be in F. It must be the smallest element of F and so f(1) = 1. Hence g(1) = 2. We can never have two successive integers n and n+1 in G, because if g(m) = n+1, then f(something) = n and so n is in F and G. Contradiction. In particular, 3 must be in F, and so f(2) = 3. 
Suppose f(n) = k. Then g(n) = f(k) + 1. So |Nf(k)+1 Ç G| = n. But |Nf(k)+1 Ç F| = k, so n + k = f(k) + 1, or f(k) = n + k - 1. Hence g(n) = n + k. So n + k + 1 must be in F and hence f(k+1) = n + k + 1. This so given the value of f for n we can find it for k and k+1. 
Using k+1 each time, we get, successively, f(2) = 3, f(4) = 6, f(7) = 11, f(12) = 19, f(20) = 32, f(33) = 53, f(54) = 87, f(88) = 142, f(143) = 231, f(232) = 375, which is not much help. Trying again with k, we get: f(3) = 4, f(4) = 6, f(6) = 9, f(9) = 14, f(14) = 22, f(22) = 35, f(35) = 56, f(56) = 90, f(90) = 145, f(145) = 234. Still not right, but we can try backing up slightly and using k+1: f(146) = 236. Still not right, we need to back up further: f(91) = 147, f(148) = 239, f(240) = 388. 
Busca Yahoo! 
O serviço de busca mais completo da Internet. O que você pensar o Yahoo! encontra.

[obm-l] IMO

2003-02-06 Por tôpico Johann Peter Gustav Lejeune Dirichlet
Problem 6
An international society has its members from six different countries. The list of members has 1978 names, numbered 1, 2, ... , 1978. Prove that there is at least one member whose number is the sum of the numbers of two members from his own country, or twice the number of a member from his own country. 
 
Solution
The trick is to use differences. 
At least 6.329 = 1974, so at least 330 members come from the same country, call it C1. Let their numbers be a1  a2  ...  a330. Now take the 329 differences a2 - a1, a3 - a1, ... , a330 - a1. If any of them are in C1, then we are home, so suppose they are all in the other five countries. 
At least 66 must come from the same country, call it C2. Write the 66 as b1  b2  ...  b66. Now form the 65 differences b2 - b1, b3 - b1, ... , b66 - b1. If any of them are in C2, then we are home. But each difference equals the difference of two of the original ais, so if it is in C1 we are also home. 
So suppose they are all in the other four countries. At least 17 must come from the same country, call it C3. Write the 17 as c1  c2  ...  c17. Now form the 16 differences c2 - c1, c3 - c1, ... , c17 - c1. If any of them are in C3, we are home. Each difference equals the difference of two bis, so if any of them are in C2 we are home. [For example, consider ci - c1. Suppose ci = bn - b1 and c1 = bm - b1, then ci - c1 = bn - bm, as claimed.]. Each difference also equals the difference of two ais, so if any of them are in C1, we are also home. [For example, consider ci - c1, as before. Suppose bn = aj - a1, bm = ak - a1, then ci - c1 = bn - bm = aj - ak, as claimed.] 
So suppose they are all in the other three countries. At least 6 must come from the same country, call it C4. We look at the 5 differences and conclude in the same way that at least 3 must come from C5. Now the 2 differences must both be in C6 and their difference must be in one of the C1, ... , C6 giving us the required sum. 
Busca Yahoo! 
O serviço de busca mais completo da Internet. O que você pensar o Yahoo! encontra.

[obm-l] IMO

2003-02-06 Por tôpico Johann Peter Gustav Lejeune Dirichlet
Problem 6
Let A and E be opposite vertices of an octagon. A frog starts at vertex A. From any vertex except E it jumps to one of the two adjacent vertices. When it reaches E it stops. Let an be the number of distinct paths of exactly n jumps ending at E. Prove that: 
   a2n-1 = 0a2n = (2 + Ö2)n-1/Ö2 - (2 - Ö2)n-1/Ö2. 
 
Solution
Each jump changes the parity of the shortest distance to E. The parity is initially even, so an odd number of jumps cannot end at E. Hence a2n-1 = 0. 
We derive a recurrence relation for a2n. This is not easy to do directly, so we introduce bn which is the number of paths length n from C to E. Then we have immediately: 
  a2n = 2a2n-2 + 2b2n-2 for n  1   b2n = 2b2n-2 + a2n-2 for n  1 
Hence, using the first equation: a2n - 2a2n-2 = 2a2n-2 - 4a2n-4 + 2b2n-2 - 4b2n-4 for n  2. Using the second equation, this leads to: a2n = 4a2n-2 - 2a2n-4 for n  2. This is a linear recurrence relation with the general solution: a2n = a(2 + Ö2)n-1 + b(2 - Ö2)n-1. But we easily see directly that a4 = 2, a6 = 8 and we can now solve for the coefficients to get the solution given. 
Busca Yahoo! 
O serviço de busca mais completo da Internet. O que você pensar o Yahoo! encontra.

[obm-l] IMO

2003-02-06 Por tôpico Johann Peter Gustav Lejeune Dirichlet
Problem 6
Show that there exists a set A of positive integers with the following property: for any infinite set S of primes, there exist two positive integers m in A and n not in A, each of which is a product of k distinct elements of S for some k = 2. 
 
Solution
Let the primes be p1  p2  p3  ... . Let A consists of all products of n distinct primes such that the smallest is greater than pn. For example: all primes except 2 are in A; 21 is not in A because it is a product of two distinct primes and the smallest is greater than 3. Now let S = {pi1, pi2, ... } be any infinite set of primes. Assume that pi1  pi2  ... . Let n = i1. Then pi1pi2 ... pin is not in A because it is a product of n distinct primes, but the smallest is not greater than pn. But pi2pi3 ... pin+1 is in A, because it is a product of n distinct primes and the smallest is greater than pn. But both numbers are products of n distinct elements of S. 
Busca Yahoo! 
O serviço de busca mais completo da Internet. O que você pensar o Yahoo! encontra.

[obm-l] IMO

2003-02-06 Por tôpico Johann Peter Gustav Lejeune Dirichlet
Problem 6
For each positive integer n, S(n) is defined as the greatest integer such that for every positive integer k = S(n), n2 can be written as the sum of k positive squares. 
(a) Prove that S(n) = n2 - 14 for each n = 4. (b) Find an integer n such that S(n) = n2 - 14. (c) Prove that there are infinitely many integers n such that S(n) = n2 - 14. 
 
Solution
(a) Let N = n2. Suppose we could express N as a sum of N - 13 squares. Let the number of 4s be a, the number of 9s be b and so on. Then we have 13 = 3a + 8b + 15c + ... . Hence c, d, ... must all be zero. But neither 13 nor 8 is a multiple of 3, so there are no solutions. Hence S(n) = N - 14. 
A little experimentation shows that the problem is getting started. Most squares cannot be expressed as a sum of two squares. For N = 132 = 169, we find: 169 = 9 + 4 + 4 + 152 1s, a sum of 155 = N - 14 squares. By grouping four 1s into a 4 repeatedly, we obtain all multiples of 3 plus 2 down to 41 (169 = 9 + 40 4s). Then grouping four 4s into a 16 gives us 38, 35, ... , 11 (169 = 10 16s + 9). Grouping four 16s into a 64 gives us 8 and 5. We obtain the last number congruent to 2 mod 3 by the decomposition: 169 = 122 + 52. 
For the numbers congruent to 1 mod 3, we start with N - 15 = 154 squares: 169 = 5 4s + 149 1s. Grouping as before gives us all 3m + 1 down to 7: 169 = 64 + 64 + 16 + 16 + 4 + 4 + 1. We may use 169 = 102 + 82 + 22 + 12 for 4. 
For multiples of 3, we start with N - 16 = 153 squares: 169 = 9 + 9 + 151 1s. Grouping as before gives us all multiples of 3 down to 9: 169 = 64 + 64 + 16 + 9 + 9 + 4 + 1 + 1 + 1. Finally, we may take 169 = 122 + 42 + 32 for 3 and split the 42 to get 169 = 122 + 32 + 22 + 22 + 22 + 22 for 6. That completes the demonstration that we can write 132 as a sum of k positive squares for all k = S(13) = 132 - 14. 
We now show how to use the expressions for 132 to derive further N. For any N, the grouping technique gives us the high k. Simply grouping 1s into 4s takes us down: from 9 + 4 + 4 + (N-17) 1s to (N-14)/4 + 6  N/2 or below; from 4 + 4 + 4 + 4 + 4 + (N-20) 1s to (N-23)/4 + 8  N/2 or below; from 9 + 9 + (N-18) 1s to (N-21)/4 + 5  N/2 or below. So we can certainly get all k in the range (N/2 to N-14) by this approach. Now suppose that we already have a complete set of expressions for N1 and for N2 (where we may have N1 = N2). Consider N3 = N1N2. Writing N3 = N1( an expression for N2 as a sum of k squares) gives N3 as a sum of 1 thru k2 squares, where k2 = N2 - 14 squares (since N1 is a square). Now express N1 as a sum of two squares: n12 + n22. We have N3 = n12(a sum of k2 squares) + n22(a sum of k squares). This gives N3 as a sum of k2 + 1 thru 2k2 squares. Continuing in this way gives N3 as a sum of 1 thru k1k2 squares. But ki = Ni - 14  2/3 Ni, so k1k2  N3/2. So when combined with the top down grouping we get a complete set of expressions for N3. 
This shows that there are infinitely many squares N with a complete set of expressions, for example we may take N = the squares of 13, 132, 133, ... . 
Busca Yahoo! 
O serviço de busca mais completo da Internet. O que você pensar o Yahoo! encontra.

Re: [obm-l] IMO

2002-08-08 Por tôpico Fabio Dias Moreira

On Wed, Aug 07, 2002 at 03:44:34AM +, Fernanda Medeiros wrote:
 Ol? pessoal, ser? q algu?m pode me dar uma ajuda nessa quest?o?! Valeu!
 F?
 (IMO)
 Considere um inteiro positivo r e um retangulo de dimens?es |AB|=20 ,
 |BC|=12.O retangulo ? dividido em uma grade de 20x12 quadrados 
 unit?rios.Uma moeda pode ser movida de um quadrado a outro = a distancia 
 entre os centros dos uqadrados ? sqrt(r). A tarefa ? encontrar uma 
 sequencia de movimentos q levem uma moeda do quadrado q tem A  com v?rtice 
 ao quadrado q tem B como v?rtice.
 a)Mostre q a tarefa n?o pode ser feita se r ? divisivel por 3 ou por 2.
 b)Prove q a tarefa pode ser feitase r=73
 c)Pode a tarefa ser feita qnd r=97?
 

(Esse problema caiu numa IMO? Quem já jogou variantes mais exóticas de xadrez[1] teve 
uma boa ajuda nesse problema.)

a) Note que r deve ser da forma a*a+b*b para a e b inteiros. Se r é divisível por 3, 
como devem ser a e b? Para o caso de 2|r, pense da mesma forma e olhe para o retângulo 
como um tabuleiro de xadrez.

b) Resolva a equação a*a+b*b=73 para achar os possíveis movimentos da moeda.

c) Idem.

Referências:

[1] http://www.chessvariants.com/

[]s,

-- 
Fábio Dias Moreira ([EMAIL PROTECTED])
GPG fingerprint: 72F8 289F 1118 D225 700E  28D9 6A53 9016 BBF3 190A



msg07727/pgp0.pgp
Description: PGP signature


RE: [obm-l] IMO dia 1, Q1 (solucao?!)

2002-07-29 Por tôpico Ralph Teixeira

Estah correto...

Mas soh para voces terem uma ideia de como o pessoal lah era rigoroso,
esta solucao valeria 6 pontos.

O pequeno detalhe que estah faltando eh o seguinte. NO caso (2),
dividimos a inducao em T_{k} e T_{n-k} e, por inducao acabou, certo? Bom,
nao exatamente... Note que poderiamos ter k=0 ou k=n, e um dos triangulos
simplesmente nao teria ponto algum. Entao estah faltando uma das duas
coisas:

(i) Ou voce cita o caso T_{0} explicitamente e nota que tambem vale a
tal proposicao (voce soh citou T_1 e T_2)...
(ii) ...ou voce separa o caso 2 em 2(a) (que vira dois triangulos) e
este caso especial (onde ha de fato um triangulo soh T_{n}).

Eles nao queriam uma demonstracao complicada destas coisas, que sao de
fato obvias. O que eles querem eh uma *mencao* de que este caso (o
triangulo vazio) existia e nao se enquadrava perfeitamente na inducao. No
criterio de correcao, nao fazer o caso T_0 era um erro mais ou menos
semelhante a esquecer o caso inicial de uma inducao... e por isso perdia-se
um ponto (o que explica a grande quantidade de 6 desta questao).

Abraco,
 Ralph


-Original Message-
From: Marcio
To: [EMAIL PROTECTED]
Sent: 7/27/02 9:18 AM
Subject: [obm-l] IMO dia 1, Q1 (solucao?!)

Me ajudem a detectar possiveis falhas nessa solucao!

Traducao : Seja n  0 inteiro.  Seja T_n o conjunto dos ptos (x,y) do
plano com x,y inteiros nao negativos e x+y  n. Cada pto de T eh pintado
de
R ou B. Se (x,y) eh R, entao tmb o serao tds os ptos (x',y') de Tcom x'
= x
e y'=y. Defina uma X-set como um conjunto de n ptos azuis com
coordenadas x
distintas, e uma
Y-set como um conjunto de n ptos azuis com coordenadas y distintas.
Prove
que o nr de X-sets eh igual ao nr de Y-sets.

Minha solucao foi por inducao na seguinte proposicao:
Se a n-upla P = (p0, p1, ..., p_(n-1) ) da a qtd de ptos pintados de B
nas
retas x=0, x=1, ..., x=n-1 (respectivamente), entao a qtd de B's nas
retas
y=0, y=1, ..., y=n-1 nessa ordem eh dada por uma permutacao de P. (em
particular nr de X-sets = nr de Y-sets = Produtorio de p_i).

Em 1o lugar, note que se (x,y)=B, entao (x', y') = B sempre que x'=x ou
y'=y.

Para n=1, n=2 eh soh considerar todos os (poucos) casos possiveis e
confirmar que eh verdade.
Suponha valido para inteiros menores ou iguais a n, e consideremos o
caso
n+1.

1) Se #X eh nao nulo, entao toda a diagonal externa x+y=n eh B (de fato,
se
(a,n-a) = R, entao todos abaixo dele sao R e nessa reta x=a nao existe
nenhum pto B).
Apagando essa diagonal, note que o que sobre eh uma configuracao valida
em
T_n e portanto, se nessa configuracao temos P = (p0, p1, ..., p_(n-1) )
B's
nas retas x=0,1,...,n-1, teremos /P = permutacao de P B's nas retas
y=0,...
Reescrevendo a diagonal soh de B's, teremos P'=(p0+1, p1+1, ..., p_(n-1)
+
1, 1) associada a qtd de ptos pintados de B nas retas x=0, x=1,... x=n e
/P'
= (elementos de /P somados de 1 unidade, com 1 no final), donde /P' eh
uma
permutacao de P'.

2) Se #X eh nulo, entao existe k tq a reta x=k soh tem R. Apagando o
retangulo de vertices
(0,0)-(k,0)-(k,n-k)-(0,n-k), ficamos com uma configuracao valida de
T_(k)
(considerada sobre um novo eixo transladado em relacao ao original e com
centro em (0, n-k+1) e outra de T_(n-k) (...centro em (k+1,0) ) nas
quais
podemos aplicar a hipotese de inducao e proceder como em (1).

Isso conclui a inducao e o problema.

Abracos,
Marcio

PS: Tmb tentei o problema 3, mas o melhor que eu consegui foi verificar
que
se a divisao vale para infinitos inteiros, entao o polinomio do
denominador
(em a) deve dividir o polinomio do numerador.. Depois devo tentar os
problemas do 2o dia..


=
Instruções para entrar na lista, sair da lista e usar a lista em
http://www.mat.puc-rio.br/~nicolau/olimp/obm-l.html
O administrador desta lista é [EMAIL PROTECTED]

=
=
Instruções para entrar na lista, sair da lista e usar a lista em
http://www.mat.puc-rio.br/~nicolau/olimp/obm-l.html
O administrador desta lista é [EMAIL PROTECTED]
=



RE: [obm-l] IMO dia 1, Q1 (solucao?!)

2002-07-29 Por tôpico Johann Dirichlet

Peguei as provas em PS e PDF da IMO.Se alguem
puder me dizercomo eu faço para escrever um
arquivo PS sendo que eu so tenho os
visualizadores. E eu consegui fazer apenas o
problema 2 desta IMO(geometria cearense sem do
nem piedade.Estilo problema 1 da IMO da Coreia.
  

--- Ralph Teixeira [EMAIL PROTECTED] escreveu: 
Estah correto...
 
 Mas soh para voces terem uma ideia de como
 o pessoal lah era rigoroso,
 esta solucao valeria 6 pontos.
 
 O pequeno detalhe que estah faltando eh o
 seguinte. NO caso (2),
 dividimos a inducao em T_{k} e T_{n-k} e, por
 inducao acabou, certo? Bom,
 nao exatamente... Note que poderiamos ter k=0
 ou k=n, e um dos triangulos
 simplesmente nao teria ponto algum. Entao estah
 faltando uma das duas
 coisas:
 
 (i) Ou voce cita o caso T_{0}
 explicitamente e nota que tambem vale a
 tal proposicao (voce soh citou T_1 e T_2)...
 (ii) ...ou voce separa o caso 2 em 2(a)
 (que vira dois triangulos) e
 este caso especial (onde ha de fato um
 triangulo soh T_{n}).
 
 Eles nao queriam uma demonstracao
 complicada destas coisas, que sao de
 fato obvias. O que eles querem eh uma *mencao*
 de que este caso (o
 triangulo vazio) existia e nao se enquadrava
 perfeitamente na inducao. No
 criterio de correcao, nao fazer o caso T_0 era
 um erro mais ou menos
 semelhante a esquecer o caso inicial de uma
 inducao... e por isso perdia-se
 um ponto (o que explica a grande quantidade de
 6 desta questao).
 
 Abraco,
  Ralph
 
 
 -Original Message-
 From: Marcio
 To: [EMAIL PROTECTED]
 Sent: 7/27/02 9:18 AM
 Subject: [obm-l] IMO dia 1, Q1 (solucao?!)
 
 Me ajudem a detectar possiveis falhas nessa
 solucao!
 
 Traducao : Seja n  0 inteiro.  Seja T_n o
 conjunto dos ptos (x,y) do
 plano com x,y inteiros nao negativos e x+y  n.
 Cada pto de T eh pintado
 de
 R ou B. Se (x,y) eh R, entao tmb o serao tds os
 ptos (x',y') de Tcom x'
 = x
 e y'=y. Defina uma X-set como um conjunto de n
 ptos azuis com
 coordenadas x
 distintas, e uma
 Y-set como um conjunto de n ptos azuis com
 coordenadas y distintas.
 Prove
 que o nr de X-sets eh igual ao nr de Y-sets.
 
 Minha solucao foi por inducao na seguinte
 proposicao:
 Se a n-upla P = (p0, p1, ..., p_(n-1) ) da a
 qtd de ptos pintados de B
 nas
 retas x=0, x=1, ..., x=n-1 (respectivamente),
 entao a qtd de B's nas
 retas
 y=0, y=1, ..., y=n-1 nessa ordem eh dada por
 uma permutacao de P. (em
 particular nr de X-sets = nr de Y-sets =
 Produtorio de p_i).
 
 Em 1o lugar, note que se (x,y)=B, entao (x',
 y') = B sempre que x'=x ou
 y'=y.
 
 Para n=1, n=2 eh soh considerar todos os
 (poucos) casos possiveis e
 confirmar que eh verdade.
 Suponha valido para inteiros menores ou iguais
 a n, e consideremos o
 caso
 n+1.
 
 1) Se #X eh nao nulo, entao toda a diagonal
 externa x+y=n eh B (de fato,
 se
 (a,n-a) = R, entao todos abaixo dele sao R e
 nessa reta x=a nao existe
 nenhum pto B).
 Apagando essa diagonal, note que o que sobre eh
 uma configuracao valida
 em
 T_n e portanto, se nessa configuracao temos P =
 (p0, p1, ..., p_(n-1) )
 B's
 nas retas x=0,1,...,n-1, teremos /P =
 permutacao de P B's nas retas
 y=0,...
 Reescrevendo a diagonal soh de B's, teremos
 P'=(p0+1, p1+1, ..., p_(n-1)
 +
 1, 1) associada a qtd de ptos pintados de B nas
 retas x=0, x=1,... x=n e
 /P'
 = (elementos de /P somados de 1 unidade, com 1
 no final), donde /P' eh
 uma
 permutacao de P'.
 
 2) Se #X eh nulo, entao existe k tq a reta x=k
 soh tem R. Apagando o
 retangulo de vertices
 (0,0)-(k,0)-(k,n-k)-(0,n-k), ficamos com uma
 configuracao valida de
 T_(k)
 (considerada sobre um novo eixo transladado em
 relacao ao original e com
 centro em (0, n-k+1) e outra de T_(n-k)
 (...centro em (k+1,0) ) nas
 quais
 podemos aplicar a hipotese de inducao e
 proceder como em (1).
 
 Isso conclui a inducao e o problema.
 
 Abracos,
 Marcio
 
 PS: Tmb tentei o problema 3, mas o melhor que
 eu consegui foi verificar
 que
 se a divisao vale para infinitos inteiros,
 entao o polinomio do
 denominador
 (em a) deve dividir o polinomio do numerador..
 Depois devo tentar os
 problemas do 2o dia..
 


 =
 Instruções para entrar na lista, sair da lista
 e usar a lista em

http://www.mat.puc-rio.br/~nicolau/olimp/obm-l.html
 O administrador desta lista é
 [EMAIL PROTECTED]


 =

=
 Instruções para entrar na lista, sair da lista
 e usar a lista em

http://www.mat.puc-rio.br/~nicolau/olimp/obm-l.html
 O administrador desta lista é
 [EMAIL PROTECTED]

= 

___
Yahoo! PageBuilder
O super editor para criação de sites: é grátis, fácil e rápido.
http://br.geocities.yahoo.com/v/pb.html

Re: [obm-l] IMO dia 1, Q2 (solucao)(comentario de JOHANN)

2002-07-29 Por tôpico Johann Dirichlet

Mas tu e um porre hein Cohen??
Pra que complexos se da pra fazer com Geometria
Cearense(marcar angulos ate se cansar)?
Essa soluçao e parecida com a do Daniel Uno para
a questao 1 da IMOP da Coreia(veja Eureka 9 no
site da OBM). 

Eu nao vou passar a soluçao integral que eu
fiz.Mas essas dicas ja dao conta do recado.
Prove que os triangulos AOF e AOE sao
equilateros.
Chame angBOE=4x e calcule todos os angulos em
funçao de x.Desenhe o ponto I incentro de CEF(que
deve ser o J,certo?).Provaremos que
angDAO=angAOI,o que acarreta o paralelismo. 
Veja que CA e bissetriz de angECF,logo I
e encontro de CA e a bissetriz de angEFC.
Agora faça o arrastao(marque tudo que e angulo)e 
prontoE so ver depois de infindaveis
contas que OAJ e isosceles.
Como brincadeira prove que EOJF e ciclico(olhe
para AO,AJ,...
 --- Marcio [EMAIL PROTECTED] escreveu:  Essa
eh para fortalecer os numeros complexos (o
 enunciado traduzido esta
 no final).
 Q2)  Vou usar a' para representar o conjugado
 de a. Os lemas abaixo sao
 usados toda hora em problemas de geometria, e
 por isso eu os coloquei em
 evidencia.
 
 1. Suponha, spg, q o circunraio de ABC eh 1.
 Ponha B=-1, C=1, A=a^2 =
 cis(2x); a = cis(x),
 com 30x90. Note que a' = 1/a, (a^2)'=1/(a^2).
 Lema1: Se x e y sao pontos do circulo unitario,
 a reta que os une tem eq. z
 + (xy)z' = x+y.
 Lema2: Os pontos medios dos arcos formados
 pelos complexos a^2 e b^2 sao ab
 e -ab (de fato, se m eh medio,
 arg(m)-arg(a)=arg(b)-arg(m) = m/a=b/m).
 
 2. Determinacao do ponto J:
 Ponto D: arg(D) = (180+2x)/2 = 90+x donde d =
 cis(90)*cisx = ia
 Reta AD: z + (ia^3)z' = ...
 Reta OJ (//AD passando pela origem):   z +
 (ia^3)z' = 0(1)
 Reta AC:   
z + (a^2)z' = a^2 + 1
 (2)
 Resolvendo as eqs (1) e (2), encontramos o
 ponto J: z = a(a-i)
 
 3. Determinando E,F:
 Temos |z-a^2|=|z| (esta na mediatriz) e zz' = 1
 (esta na circunferencia),
 logo
 (z-a^2)(z' - 1/a^2) = zz'
 Simplificando, z^2 - (a^2)z + a^4 = 0.
 Usando baskara ou multiplicando os 2 lados por
 z+a^2, obtemos p. ex:
 e = (a^2)cis(60)  e
 f = (a^2)cis(-60)
 
 4. Ponto medio do arco CF:
 m=acis(-30)
 (note que, como x30, esse ponto eh de fato o
 que esta entre C e F, pois
 argm = x-300).
 
 5. J esta na bissetriz EM:
 Eh soh ver que J verifica a eq. da reta EM: z +
 (a^3)cis(30)z' = (a^2)cis60
 + acis(-30) .
 Eh soh substituir z=a(a-i), z'=(1+ia)/(a^2) e
 ver que os coeficientes de a e
 de a^2 sao iguais dos dois lados.
 
 6. J esta na bissetriz de C:
 O pto medio do arco EF que nao contem C eh
 sqrt(e*f) = a^2 = A. Logo, a
 bissetriz de C eh exatamente a reta CA, donde J
 esta nessa bissetriz. (essa
 parte ateh eu consegui fazer por plana :)
 
 7. Logo, J pertence a duas bissetrizes, e
 portanto eh o incentro.
 
 Estive tentando fazer as questoes do primeiro
 dia da prova.. Comecei pela 2,
 que achei mais facil, e depois tentei a
 primeira.. parei de tentar a 3 agora
 pq nao estava produzindo muita coisa.. Nos
 proximos dois emails vou mandar
 minhas ideias/solucoes.. Mandem as suas tmb!
 
 Abracos,
 Marcio
 
 - Original Message -
 From: Ralph Teixeira [EMAIL PROTECTED]
 To: 'Rodrigo Villard Milet '
 [EMAIL PROTECTED]; 'Obm '
 [EMAIL PROTECTED]
 Sent: Friday, July 26, 2002 10:10 AM
 Subject: RE: [obm-l] IMO!?!?
 
 
  Let \ $BC$ be a diameter of the circle
 ${\Gamma}$ with
  centre $O$. \ Let $A$ be a point on $\Gamma$
 such that $0{{}^\circ
  }\angle AOB120{{}^\circ}$. \ Let $D$ be the
 midpoint of the
  arc $AB$ not containing $C$. \ The line
 through $O$ parallel
  to $DA$ meets the line $AC$ at $J$. \ The
 perpendicular bisector
  of $OA$ meets $\Gamma$ at $E$ and at $F$. \
 Prove that $J$ is
  the incentre of the triangle $CEF$.
 
 Traducao: Seja BC diametro de um
 circunferencia com centro O. Seja A um
 pto da circunferencia com AOB120 graus. Seja D
 medio do arco AB que nao
 contem C. A reta por O paralela a DA encontra
 AC em J. A mediatriz de OA
 encontra a circunferencia em E e F. Mostre que
 J eh incentro do triangulo
 CEF.
 
 

=
 Instruções para entrar na lista, sair da lista
 e usar a lista em

http://www.mat.puc-rio.br/~nicolau/olimp/obm-l.html
 O administrador desta lista é
 [EMAIL PROTECTED]

= 

___
Yahoo! PageBuilder
O super editor para criação de sites: é grátis, fácil e rápido.
http://br.geocities.yahoo.com/v/pb.html
=
Instruções para entrar na lista, sair da lista e usar a lista em
http://www.mat.puc-rio.br/~nicolau/olimp/obm-l.html
O administrador desta lista é [EMAIL PROTECTED]
=



RE: [obm-l] IMO dia 1, Q1 (solucao?!)

2002-07-29 Por tôpico Johann Dirichlet

 --- Johann Dirichlet
[EMAIL PROTECTED] escreveu:  Peguei
as provas em PS e PDF da IMO.Se alguem
 puder me dizercomo eu faço para escrever um
 arquivo PS sendo que eu so tenho os
 visualizadores. E eu consegui fazer apenas o
 problema 2 desta IMO(geometria cearense sem do
 nem piedade.Estilo problema 1 da IMO da Coreia.
   Ao contrario do Cohen.Mas afinal de onde e
que ele teve a ideia de tirar complexos ali?
 
 --- Ralph Teixeira [EMAIL PROTECTED] escreveu:   
  
 Estah correto...
  
  Mas soh para voces terem uma ideia de
 como
  o pessoal lah era rigoroso,
  esta solucao valeria 6 pontos.
  
  O pequeno detalhe que estah faltando eh o
  seguinte. NO caso (2),
  dividimos a inducao em T_{k} e T_{n-k} e, por
  inducao acabou, certo? Bom,
  nao exatamente... Note que poderiamos ter k=0
  ou k=n, e um dos triangulos
  simplesmente nao teria ponto algum. Entao
 estah
  faltando uma das duas
  coisas:
  
  (i) Ou voce cita o caso T_{0}
  explicitamente e nota que tambem vale a
  tal proposicao (voce soh citou T_1 e T_2)...
  (ii) ...ou voce separa o caso 2 em 2(a)
  (que vira dois triangulos) e
  este caso especial (onde ha de fato um
  triangulo soh T_{n}).
  
  Eles nao queriam uma demonstracao
  complicada destas coisas, que sao de
  fato obvias. O que eles querem eh uma
 *mencao*
  de que este caso (o
  triangulo vazio) existia e nao se
 enquadrava
  perfeitamente na inducao. No
  criterio de correcao, nao fazer o caso T_0
 era
  um erro mais ou menos
  semelhante a esquecer o caso inicial de uma
  inducao... e por isso perdia-se
  um ponto (o que explica a grande quantidade
 de
  6 desta questao).
  
  Abraco,
   Ralph
  
  
  -Original Message-
  From: Marcio
  To: [EMAIL PROTECTED]
  Sent: 7/27/02 9:18 AM
  Subject: [obm-l] IMO dia 1, Q1 (solucao?!)
  
  Me ajudem a detectar possiveis falhas nessa
  solucao!
  
  Traducao : Seja n  0 inteiro.  Seja T_n o
  conjunto dos ptos (x,y) do
  plano com x,y inteiros nao negativos e x+y 
 n.
  Cada pto de T eh pintado
  de
  R ou B. Se (x,y) eh R, entao tmb o serao tds
 os
  ptos (x',y') de Tcom x'
  = x
  e y'=y. Defina uma X-set como um conjunto de
 n
  ptos azuis com
  coordenadas x
  distintas, e uma
  Y-set como um conjunto de n ptos azuis com
  coordenadas y distintas.
  Prove
  que o nr de X-sets eh igual ao nr de Y-sets.
  
  Minha solucao foi por inducao na seguinte
  proposicao:
  Se a n-upla P = (p0, p1, ..., p_(n-1) ) da a
  qtd de ptos pintados de B
  nas
  retas x=0, x=1, ..., x=n-1 (respectivamente),
  entao a qtd de B's nas
  retas
  y=0, y=1, ..., y=n-1 nessa ordem eh dada por
  uma permutacao de P. (em
  particular nr de X-sets = nr de Y-sets =
  Produtorio de p_i).
  
  Em 1o lugar, note que se (x,y)=B, entao (x',
  y') = B sempre que x'=x ou
  y'=y.
  
  Para n=1, n=2 eh soh considerar todos os
  (poucos) casos possiveis e
  confirmar que eh verdade.
  Suponha valido para inteiros menores ou
 iguais
  a n, e consideremos o
  caso
  n+1.
  
  1) Se #X eh nao nulo, entao toda a diagonal
  externa x+y=n eh B (de fato,
  se
  (a,n-a) = R, entao todos abaixo dele sao R e
  nessa reta x=a nao existe
  nenhum pto B).
  Apagando essa diagonal, note que o que sobre
 eh
  uma configuracao valida
  em
  T_n e portanto, se nessa configuracao temos P
 =
  (p0, p1, ..., p_(n-1) )
  B's
  nas retas x=0,1,...,n-1, teremos /P =
  permutacao de P B's nas retas
  y=0,...
  Reescrevendo a diagonal soh de B's, teremos
  P'=(p0+1, p1+1, ..., p_(n-1)
  +
  1, 1) associada a qtd de ptos pintados de B
 nas
  retas x=0, x=1,... x=n e
  /P'
  = (elementos de /P somados de 1 unidade, com
 1
  no final), donde /P' eh
  uma
  permutacao de P'.
  
  2) Se #X eh nulo, entao existe k tq a reta
 x=k
  soh tem R. Apagando o
  retangulo de vertices
  (0,0)-(k,0)-(k,n-k)-(0,n-k), ficamos com uma
  configuracao valida de
  T_(k)
  (considerada sobre um novo eixo transladado
 em
  relacao ao original e com
  centro em (0, n-k+1) e outra de T_(n-k)
  (...centro em (k+1,0) ) nas
  quais
  podemos aplicar a hipotese de inducao e
  proceder como em (1).
  
  Isso conclui a inducao e o problema.
  
  Abracos,
  Marcio
  
  PS: Tmb tentei o problema 3, mas o melhor que
  eu consegui foi verificar
  que
  se a divisao vale para infinitos inteiros,
  entao o polinomio do
  denominador
  (em a) deve dividir o polinomio do
 numerador..
  Depois devo tentar os
  problemas do 2o dia..
  
 


  =
  Instruções para entrar na lista, sair da
 lista
  e usar a lista em
 

http://www.mat.puc-rio.br/~nicolau/olimp/obm-l.html
  O administrador desta lista é
  [EMAIL PROTECTED]
 


  =
 

=
  Instruções para entrar na lista, sair da
 lista
  e usar a lista em
 

http://www.mat.puc-rio.br/~nicolau/olimp/obm-l.html
  O administrador desta lista é

[obm-l] IMO dia 1, Q2 (solucao)

2002-07-27 Por tôpico Marcio

Essa eh para fortalecer os numeros complexos (o enunciado traduzido esta
no final).
Q2)  Vou usar a' para representar o conjugado de a. Os lemas abaixo sao
usados toda hora em problemas de geometria, e por isso eu os coloquei em
evidencia.

1. Suponha, spg, q o circunraio de ABC eh 1. Ponha B=-1, C=1, A=a^2 =
cis(2x); a = cis(x),
com 30x90. Note que a' = 1/a, (a^2)'=1/(a^2).
Lema1: Se x e y sao pontos do circulo unitario, a reta que os une tem eq. z
+ (xy)z' = x+y.
Lema2: Os pontos medios dos arcos formados pelos complexos a^2 e b^2 sao ab
e -ab (de fato, se m eh medio, arg(m)-arg(a)=arg(b)-arg(m) = m/a=b/m).

2. Determinacao do ponto J:
Ponto D: arg(D) = (180+2x)/2 = 90+x donde d = cis(90)*cisx = ia
Reta AD: z + (ia^3)z' = ...
Reta OJ (//AD passando pela origem):   z + (ia^3)z' = 0(1)
Reta AC:   z + (a^2)z' = a^2 + 1
(2)
Resolvendo as eqs (1) e (2), encontramos o ponto J: z = a(a-i)

3. Determinando E,F:
Temos |z-a^2|=|z| (esta na mediatriz) e zz' = 1 (esta na circunferencia),
logo
(z-a^2)(z' - 1/a^2) = zz'
Simplificando, z^2 - (a^2)z + a^4 = 0.
Usando baskara ou multiplicando os 2 lados por z+a^2, obtemos p. ex:
e = (a^2)cis(60)  e
f = (a^2)cis(-60)

4. Ponto medio do arco CF:
m=acis(-30)
(note que, como x30, esse ponto eh de fato o que esta entre C e F, pois
argm = x-300).

5. J esta na bissetriz EM:
Eh soh ver que J verifica a eq. da reta EM: z + (a^3)cis(30)z' = (a^2)cis60
+ acis(-30) .
Eh soh substituir z=a(a-i), z'=(1+ia)/(a^2) e ver que os coeficientes de a e
de a^2 sao iguais dos dois lados.

6. J esta na bissetriz de C:
O pto medio do arco EF que nao contem C eh sqrt(e*f) = a^2 = A. Logo, a
bissetriz de C eh exatamente a reta CA, donde J esta nessa bissetriz. (essa
parte ateh eu consegui fazer por plana :)

7. Logo, J pertence a duas bissetrizes, e portanto eh o incentro.

Estive tentando fazer as questoes do primeiro dia da prova.. Comecei pela 2,
que achei mais facil, e depois tentei a primeira.. parei de tentar a 3 agora
pq nao estava produzindo muita coisa.. Nos proximos dois emails vou mandar
minhas ideias/solucoes.. Mandem as suas tmb!

Abracos,
Marcio

- Original Message -
From: Ralph Teixeira [EMAIL PROTECTED]
To: 'Rodrigo Villard Milet ' [EMAIL PROTECTED]; 'Obm '
[EMAIL PROTECTED]
Sent: Friday, July 26, 2002 10:10 AM
Subject: RE: [obm-l] IMO!?!?


 Let \ $BC$ be a diameter of the circle ${\Gamma}$ with
 centre $O$. \ Let $A$ be a point on $\Gamma$ such that $0{{}^\circ
 }\angle AOB120{{}^\circ}$. \ Let $D$ be the midpoint of the
 arc $AB$ not containing $C$. \ The line through $O$ parallel
 to $DA$ meets the line $AC$ at $J$. \ The perpendicular bisector
 of $OA$ meets $\Gamma$ at $E$ and at $F$. \ Prove that $J$ is
 the incentre of the triangle $CEF$.

Traducao: Seja BC diametro de um circunferencia com centro O. Seja A um
pto da circunferencia com AOB120 graus. Seja D medio do arco AB que nao
contem C. A reta por O paralela a DA encontra AC em J. A mediatriz de OA
encontra a circunferencia em E e F. Mostre que J eh incentro do triangulo
CEF.


=
Instruções para entrar na lista, sair da lista e usar a lista em
http://www.mat.puc-rio.br/~nicolau/olimp/obm-l.html
O administrador desta lista é [EMAIL PROTECTED]
=



[obm-l] IMO dia 1, Q1 (solucao?!)

2002-07-27 Por tôpico Marcio

Me ajudem a detectar possiveis falhas nessa solucao!

Traducao : Seja n  0 inteiro.  Seja T_n o conjunto dos ptos (x,y) do
plano com x,y inteiros nao negativos e x+y  n. Cada pto de T eh pintado de
R ou B. Se (x,y) eh R, entao tmb o serao tds os ptos (x',y') de Tcom x' = x
e y'=y. Defina uma X-set como um conjunto de n ptos azuis com coordenadas x
distintas, e uma
Y-set como um conjunto de n ptos azuis com coordenadas y distintas. Prove
que o nr de X-sets eh igual ao nr de Y-sets.

Minha solucao foi por inducao na seguinte proposicao:
Se a n-upla P = (p0, p1, ..., p_(n-1) ) da a qtd de ptos pintados de B nas
retas x=0, x=1, ..., x=n-1 (respectivamente), entao a qtd de B's nas retas
y=0, y=1, ..., y=n-1 nessa ordem eh dada por uma permutacao de P. (em
particular nr de X-sets = nr de Y-sets = Produtorio de p_i).

Em 1o lugar, note que se (x,y)=B, entao (x', y') = B sempre que x'=x ou
y'=y.

Para n=1, n=2 eh soh considerar todos os (poucos) casos possiveis e
confirmar que eh verdade.
Suponha valido para inteiros menores ou iguais a n, e consideremos o caso
n+1.

1) Se #X eh nao nulo, entao toda a diagonal externa x+y=n eh B (de fato, se
(a,n-a) = R, entao todos abaixo dele sao R e nessa reta x=a nao existe
nenhum pto B).
Apagando essa diagonal, note que o que sobre eh uma configuracao valida em
T_n e portanto, se nessa configuracao temos P = (p0, p1, ..., p_(n-1) ) B's
nas retas x=0,1,...,n-1, teremos /P = permutacao de P B's nas retas y=0,...
Reescrevendo a diagonal soh de B's, teremos P'=(p0+1, p1+1, ..., p_(n-1) +
1, 1) associada a qtd de ptos pintados de B nas retas x=0, x=1,... x=n e /P'
= (elementos de /P somados de 1 unidade, com 1 no final), donde /P' eh uma
permutacao de P'.

2) Se #X eh nulo, entao existe k tq a reta x=k soh tem R. Apagando o
retangulo de vertices
(0,0)-(k,0)-(k,n-k)-(0,n-k), ficamos com uma configuracao valida de T_(k)
(considerada sobre um novo eixo transladado em relacao ao original e com
centro em (0, n-k+1) e outra de T_(n-k) (...centro em (k+1,0) ) nas quais
podemos aplicar a hipotese de inducao e proceder como em (1).

Isso conclui a inducao e o problema.

Abracos,
Marcio

PS: Tmb tentei o problema 3, mas o melhor que eu consegui foi verificar que
se a divisao vale para infinitos inteiros, entao o polinomio do denominador
(em a) deve dividir o polinomio do numerador.. Depois devo tentar os
problemas do 2o dia..

=
Instruções para entrar na lista, sair da lista e usar a lista em
http://www.mat.puc-rio.br/~nicolau/olimp/obm-l.html
O administrador desta lista é [EMAIL PROTECTED]
=



Re: [obm-l] IMO dia 2, Q4 (solucao)

2002-07-27 Por tôpico Marcio

Essa eh a minha solucao para o problema 4, do 2o dia.. O 5 eu tentei mas nao
consigui progredir muito.. E o 6 eu nem tive coragem de tentar escrever..

Traducao: Seja n um inteiro maior que 1. Os divisores positivos de n sao
d1, d2, ..., dk; onde
1=d1d2d3...dk=n.
Defina D =d1*d2 + d2*d3 + d3*d4 + ... + dk-1*dk.
a) Mostre que D  n^2
b) Determine todos os n para os quais D eh divisor de n^2.

Engracado que esse exercicio me pareceu no mesmo estilo e mesmo nivel de
dificuldade que o 4 da imo2001... Talvez a maior dica seja o fato de as
letras (a) e (b) a principio parecerem nao ter nada a ver uma com a outra...

a) Eh facil ver que d_(k-1) = n /d2 (se fosse da forma n/di com i2,
teriamos did2 e n/d2n/di), e em geral, d_(k-a) = n / d_(a+1).

1) se n nao for quadrado perfeito
Juntando os extremos (lembrando que se n nao for quadrado, D tem um numero
impar (k-1) de parcelas, e o termo do meio nao sera juntado),
temos pela desigualdade das medias que:
d1d2 + d_(k-1)*dk = 2n,
d2d3 + d_(k-2)*d_(k-1) = 2n
...
termo do meio: d_(k-1)/2 * d_(k+1)/2 = n

Somando tudo, temos D = [(k-2)/2]*2n + n  n(n-2)+n = n^2 - n  n^2.
(claro que k=n, pois todo divisor de n eh menor ou igual a n e soh
existem n naturais = n)
2) Se n for quadrado perfeito, nao existe parcela do meio.. Eh soh somar as
k-1 desigualdades para obter
D = [(k-1)/2]*2n = (n-1)*n = n^2 - n  n^2.

b) A letra (a) eh a dica..
Testando um pouco, nota-se que se n for primo, entao D=1.n = n eh um divisor
de n^2.
Se porem n nao for primo, entao seus divisores sao 1d2...= d_(k-1)n, com
d2 diferente de n
(pd se ter d2=d_(k-1) se n for quadrado).
Portanto, D = 1.d2 + ... + d_(k-1)*n = 1.d2 + ... + n^2 / d2  n^2 / d2.

Logo, pela letra (a), temos (n^2 / d2)  D  n^2 e isso implica que D nao eh
um divisor de n^2, pois o 2o maior divisor de n^2 eh (n^2 / d2)...  (*)
Concluimos entao que apenas os primos tem a propriedade (b).

*: de fato, td divisor de n^2 eh da forma n^2 / k, e se n^2 / k  n^2 / d2,
entao k  d2.. por outro lado, pegando um dos fatores primos p de k, temos
pkd2 e p eh um divisor de n, o que eh absurdo

Abracos,
Marcio

=
Instruções para entrar na lista, sair da lista e usar a lista em
http://www.mat.puc-rio.br/~nicolau/olimp/obm-l.html
O administrador desta lista é [EMAIL PROTECTED]
=



Re: [obm-l] IMO dia 2, Q4 (solucao corrigida)

2002-07-27 Por tôpico Marcio

Acabei de ver uma grande falha na minha letra (a)..  Usei a desigualdade das
medias ao contrario :)) A letra (b) parece estar certa...
Uma maneira correta de fazer a (a) poderia ser continuando o que eu fiz:
Eh facil ver que d_(k-1) = n /d2 (se fosse da forma n/di com i2,
teriamos did2 e n/d2n/di), e em geral, d_(j) = n / d_(k+1-j).
Mas d_(i) = i para todo i.. Logo, d_(j) = n/(k+1-j) e d_(j) * d_(j+1) =
n^2 / [(k+1-j)*(k+2-j)]
Somando em j, a soma eh telescopica (escrevendo 1/b(b+1) como 1/b - 1/(b+1))
e vemos que o lado direito eh n^2(1-1/k)  n^2...

Desculpem a besteira do ultimo email..

t+
Marcio



- Original Message -
From: Marcio [EMAIL PROTECTED]
To: [EMAIL PROTECTED]
Sent: Saturday, July 27, 2002 1:43 PM
Subject: Re: [obm-l] IMO dia 2, Q4 (solucao)


 Essa eh a minha solucao para o problema 4, do 2o dia.. O 5 eu tentei mas
nao
 consigui progredir muito.. E o 6 eu nem tive coragem de tentar escrever..

 Traducao: Seja n um inteiro maior que 1. Os divisores positivos de n sao
 d1, d2, ..., dk; onde
 1=d1d2d3...dk=n.
 Defina D =d1*d2 + d2*d3 + d3*d4 + ... + dk-1*dk.
 a) Mostre que D  n^2
 b) Determine todos os n para os quais D eh divisor de n^2.

 Engracado que esse exercicio me pareceu no mesmo estilo e mesmo nivel de
 dificuldade que o 4 da imo2001... Talvez a maior dica seja o fato de as
 letras (a) e (b) a principio parecerem nao ter nada a ver uma com a
outra...

 a) Eh facil ver que d_(k-1) = n /d2 (se fosse da forma n/di com i2,
 teriamos did2 e n/d2n/di), e em geral, d_(k-a) = n / d_(a+1).

 1) se n nao for quadrado perfeito
 Juntando os extremos (lembrando que se n nao for quadrado, D tem um numero
 impar (k-1) de parcelas, e o termo do meio nao sera juntado),
 temos pela desigualdade das medias que:
 d1d2 + d_(k-1)*dk = 2n,
 d2d3 + d_(k-2)*d_(k-1) = 2n
 ...
 termo do meio: d_(k-1)/2 * d_(k+1)/2 = n

 Somando tudo, temos D = [(k-2)/2]*2n + n  n(n-2)+n = n^2 - n  n^2.
 (claro que k=n, pois todo divisor de n eh menor ou igual a n e soh
 existem n naturais = n)
 2) Se n for quadrado perfeito, nao existe parcela do meio.. Eh soh somar
as
 k-1 desigualdades para obter
 D = [(k-1)/2]*2n = (n-1)*n = n^2 - n  n^2.

 b) A letra (a) eh a dica..
 Testando um pouco, nota-se que se n for primo, entao D=1.n = n eh um
divisor
 de n^2.
 Se porem n nao for primo, entao seus divisores sao 1d2...= d_(k-1)n,
com
 d2 diferente de n
 (pd se ter d2=d_(k-1) se n for quadrado).
 Portanto, D = 1.d2 + ... + d_(k-1)*n = 1.d2 + ... + n^2 / d2  n^2 / d2.

 Logo, pela letra (a), temos (n^2 / d2)  D  n^2 e isso implica que D nao
eh
 um divisor de n^2, pois o 2o maior divisor de n^2 eh (n^2 / d2)...  (*)
 Concluimos entao que apenas os primos tem a propriedade (b).

 *: de fato, td divisor de n^2 eh da forma n^2 / k, e se n^2 / k  n^2 /
d2,
 entao k  d2.. por outro lado, pegando um dos fatores primos p de k, temos
 pkd2 e p eh um divisor de n, o que eh absurdo

 Abracos,
 Marcio

 =
 Instruções para entrar na lista, sair da lista e usar a lista em
 http://www.mat.puc-rio.br/~nicolau/olimp/obm-l.html
 O administrador desta lista é [EMAIL PROTECTED]
 =


=
Instruções para entrar na lista, sair da lista e usar a lista em
http://www.mat.puc-rio.br/~nicolau/olimp/obm-l.html
O administrador desta lista é [EMAIL PROTECTED]
=



[obm-l] IMO, dia 2, Q5 (solução)

2002-07-27 Por tôpico Rodrigo Villard Milet




Segue a minha soluo para 
a quinta questo dessa IMO. Confiram :),( se algum tiver 
pacincia ). (f(x)+f(z))*(f(y)+f(t)) = f(xy-zt) + f(xt+yz)
Primeiramente faa x=z=0 : 2f(0) * ( f(y) + 
f(t) ) = 2f(0), logo ou f(0)=0, ou f(y)+f(t) = 1, para todos y,t reais e em 
particular quando y=t, temos f(y)=1/2, para todo y real, o que  uma 
soluo particular. Assuma ento f(0)=0.
Faa z=t=0 : f(xy)=f(x)*f(y). Ento, 
fazendo y=1, f(x)=f(x)*f(1), logo temos f(1)=1 ou f(x)=0, para todo x, o que 
 outra soluo particular. Faa y=t=1 e x=0 : 2f(z) = f(-z)+f(z), logo f  par. 
Ento, precisamos nos preocupar apenas com a parte positiva.
Na equao inicial, temos : 
f(xy)+f(xt)+f(yz)+f(zt) = f(xy-zt) + f(xt+yz), ento 
f(a)+f(b)+f(c)+f(d)=f(a-d)+f(b+c), desde que ad=bc. Colocando a=c=mx, b=d=x , 
temos 2f(mx)+2f(x)=f((m-1)x) + f((m+1)x), o que prova por induo 
que f(mx)=m^2 * f(x), para todo m inteiro. Logo f(m)=m^2, para todo m inteiro. 
f(p/q)=f(p)/f(q), pois  multiplicativa, logo f(p/q)=(p/q)^2, 
ento f(m)=m^2, para todo m racional. Vamos mostrar que f  
montona crescente em R+. Faa y=x,t=z : (f(x)+f(z))^2 = 
f(x^2-z^2)+f(2xy). Faa y=z,t=x : (f(x)+f(z))^2=f(x^2+z^2). Juntando as 
duas ltimas, temos que f(a)=f(b)+f(c), quando a^2 = b^2+c^2, logo f 
 crescente (pois se ab0, ento existe c0 tal que 
a^2=b^2+c^2, logo f(a)=f(b)+f(c) f(b), pois f(c)=f(sqrt(c))^2=0. Em 
particular f  positiva.).
Como f  crescente em R+ e f(m)=m^2 nos 
racionais, ento  fcil mostrar que f(x)=x^2 para todos x 
em R+, logo para todos x em R, pois f  par. Sejam a(n) e b(n) duas sequncias de racionais convergindo para um 
irracional x0, tais que 0a(n) x b(n). Ento como f 
 montona crescente, a(n)^2 = f(x) = b(n)^2 e fazendo n 
tender a infinito, temos a(n) e b(n) tendendo a x, logo f(x)=x^2 tambm 
nos irracionais.
Resposta : f(x)==0 ; f(x)==1/2 ; f(x)==x^2 
.

Abraos, 
Villard


Re: [obm-l] IMO!?!?

2002-07-26 Por tôpico Eduardo Casagrande Stabel


From: Ralph Teixeira [EMAIL PROTECTED]


 {\bf Problem 1}\par\nobreak

 Let $n$ be a positive integer. \ Let $T$ be the set of
 points $(x,y)$ in the plane where $x$ and $y$ are non-negative
 integers and $x+yn$. \ Each point of $T$ is coloured red or
 blue. \ If a point $(x,y)$ is red, then so are all points $(x',y')$
 of $T$ with both $ $$x'\leq x$ and $y'\leq y$. \ Define an $X$-set
 to be a set of $n$ blue points having distinct $x$-coordinates,
 and a $Y$-set to be a set of $n$ blue points having distinct
 $y$-coordinates. \ Prove that the number of $X$-sets is equal
 to the number of $Y$-sets.

Oi Pessoal,
acho que ja da pra discutir as questões...
Eu não compreendi o enunciado dessa primeira.
A gente pinta todos os pontos de T de azul ou vermelho como diz o enunciado.
Destacamos (escolhemos) n pontos dessa configuração que possuam coordenadas
distintas de x e dizemos que esse é um X-conjunto, semelhantemente
destacando n pontos dessa configuração que possuam coordenadas distintas de
y dizemos que esses n pontos formam um Y conjunto. Isso é a minha
interpretação do enunciado, está certa?
SE for isso a questão é muito fácil. Dada uma configuração determinada de
pontos e um X-conjunto nessa configuração, tomamos uma outra configuração
que consiste em inverter as cores de (x,y) pelas do ponto (y,x). Dessa forma
os pontos que formavam o X-conjunto agora formam um Y-conjunto. Portanto #
X-conjunto = # Y-conjunto, de modo análogo # Y-conjunto = # X-conjunto, e
daí # X-conjunto = # Y-conjunto.
É isso mesmo?

Eduardo.

=
Instruções para entrar na lista, sair da lista e usar a lista em
http://www.mat.puc-rio.br/~nicolau/olimp/obm-l.html
O administrador desta lista é [EMAIL PROTECTED]
=



Re: [obm-l] IMO!?!?

2002-07-26 Por tôpico David Turchick

Oi Pessoal,
acho que ja da pra discutir as questões...
Eu não compreendi o enunciado dessa primeira.
A gente pinta todos os pontos de T de azul ou vermelho como diz o 
enunciado.
Destacamos (escolhemos) n pontos dessa configuração que possuam coordenadas
distintas de x e dizemos que esse é um X-conjunto, semelhantemente
destacando n pontos dessa configuração que possuam coordenadas distintas de
y dizemos que esses n pontos formam um Y conjunto. Isso é a minha
interpretação do enunciado, está certa?

Acho que não. A tradução de define an X-set to be a set of n blue points 
having distinct x-coordinates é defina um X-conjunto como sendo um 
conjunto de n pontos azuis cujas abscissas são todas distintas entre si, e 
análogo prum Y-conjunto.

SE for isso a questão é muito fácil. Dada uma configuração determinada de
pontos e um X-conjunto nessa configuração, tomamos uma outra configuração

Tb acho que não. P/ mim, a configuração está fixada desde o começo.

David

que consiste em inverter as cores de (x,y) pelas do ponto (y,x). Dessa 
forma
os pontos que formavam o X-conjunto agora formam um Y-conjunto. Portanto #
X-conjunto = # Y-conjunto, de modo análogo # Y-conjunto = # X-conjunto, e
daí # X-conjunto = # Y-conjunto.
É isso mesmo?

Eduardo.

=
Instruções para entrar na lista, sair da lista e usar a lista em
http://www.mat.puc-rio.br/~nicolau/olimp/obm-l.html
O administrador desta lista é [EMAIL PROTECTED]
=




_
Converse com seus amigos online, faça o download grátis do MSN Messenger: 
http://messenger.msn.com.br

=
Instruções para entrar na lista, sair da lista e usar a lista em
http://www.mat.puc-rio.br/~nicolau/olimp/obm-l.html
O administrador desta lista é [EMAIL PROTECTED]
=



[obm-l] IMO!?!?

2002-07-24 Por tôpico Rodrigo Villard Milet




Onde eu acho a prova da imo de hj ?!? Se 
algum j tiver, por favor mande para a lista.
Obrigado !

Villard


[obm-l] IMO 2002

2002-05-21 Por tôpico Nicolau C. Saldanha


A equipe Brasileira que participará da IMO-2002
(19 a 30 de julho de 2002, Glasgow - UK) é a seguinte:

Líder da delegação: Prof. Edmilson Motta (São Paulo-SP)
Vice-líder da delegação: Prof. Ralph Teixeira (Niterói-RJ)

Equipe (em ordem alfabética):

BRA1: Alex Correa Abreu (Niterói-RJ)
BRA2: Davi Maximo Alexandrino Nogueira (Fortaleza-CE)
BRA3: Guilherme Issao Camarinha Fujiwara (São Paulo-SP)
BRA4: Larissa Cavalcante Queiroz de Lima (Fortaleza-CE)
BRA5: Thiago da Silva Sobral (Fortaleza-CE)
BRA6: Yuri Gomes Lima (Fortaleza-CE)

Boa sorte para nossos representantes. []s, N.
=
Instruções para entrar na lista, sair da lista e usar a lista em
http://www.mat.puc-rio.br/~nicolau/olimp/obm-l.html
O administrador desta lista é [EMAIL PROTECTED]
=